You are on page 1of 77

PAPER – 4 : COST ACCOUNTING AND FINANCIAL MANAGEMENT

PART – I : COST ACCOUNTING

QUESTIONS

Material : EOQ
1. (i) The average annual consumption of a material is 18,250 units at a price of Rs.36.50
per unit. The storage cost is 20% on an average inventory and the cost of placing
an order is Rs.50. How much quantity is to be purchased at a time?

Labour : Halsey plan


(ii) The time allowed for a job is 8 hours. The hourly rate is Rs.8. Prepare a statement
showing :
(a) The bonus earned
(b) The total earnings of labour and
(c) Hourly earnings.
Under the Halsey System with 50% bonus for time saved and Rowan System for
each hour saved progressively.

Standard Costing
(iii) If activity ratio of a company is 104% and its capacity ratio is 96%, find out its
efficiency ratio.

Material : Inventory Turnover Ratio


(iv) The following data are available in respect of material X for the year ended 31 st
March, 2008:
Opening Stock Rs.90,000
Purchases during the year 2,70,000
Closing stock 1,10,000

Calculate: (i) Inventory turnover ratio; and (ii) the number of days for which the
average inventory is held.

Operating Costing
(v) A lorry starts with a load of 20 tonnes of gods from station A. It unloads 8 tonnes at
station B and rest of goods at station C. It reaches back directly to station A after

1
getting reloaded with 16 tonnes of goods at station C. The distance between A to
B, B to C and then C to A are 80 kms., 120 kms., and 160 Kms. respectively.
Compute ‘Absolute tonnes-kms’ and ‘Commercial tonnes – kms.’

Labour : Rate per hour and wages


(vi) An employee working under a bonus scheme saves in a job for which the standard
time is 60 hours. Calculate the rate per hour worked and wages payable to a
worker if incentive bonus of 10% on hourly rate is payable when standard time,
namely 100% efficiency is achieved, and a further incentive bonus of 1% on hourly
rate for wach 1% in excess of that 100% efficiency is payable.
Assume that the normal payment is Rs.5 per hour.

Marginal Costing : Margin of Safety


(vii). A company earned a profit of Rs. 30,000 during the year 2007-08. if the marginal
cost and selling price of a product are Rs. 8 and Rs. 10 per unit respectively, find
out the amount of ‘Margin of Safety’.

Application of Marginal Costing and Flexible Budget


(viii) The ratio of variable cost to sales is 70%. The break-even point occurs at 60% of
the capacity sales. Find the capacity sales when fixed costs are Rs.90,000. Also
compute profit at 75% of the capacity sales.

Marginal Costing : Break – Even point


(ix) A company sells its product at Rs. 15 per unit. In a period, if it produces and sells
8,000 units, it incurs a loss of Rs. 5 per unit. If the volume is raised to 20,000 units,
it earns a profit of Rs. 4 per unit. Calculate break-even point both in terms of
rupees as well as in units.

Process Costing
(x) In Akanksha ltd raw material passes through four processes A, B,C and D and the
output of each process is the input of the subsequent process. The loss in the four
processes A,B,Cand D are respectively 25%, 20%, 20% and 16 ⅔ % of the Input. If
the end product at the end of process D is 40,000 kgs, what is the quantity of raw
materials required along with its cost to be fed at the beginning of Process A when
the cost of the same is Rs.5 per kg.
Introduction to Cost Accounting

2. Explain the following terms:


(i) Direct Cost

2
(ii) Marginal Cost
(iii) Responsibility Centre
(v) Cost absorption
(v) Imputed costs
(vi) Out -of- Pocket Cost
(vii) Shut Down Costs
(viii) Sunk costs
(ix) Discretionary Costs
(x) Engineered costs
Material : FIFO/LIFO
3. The following information is provided by ATLANTA Industries with regard to the
operations of raw material stores for the fortnight of April, 2007:
Material ASH:
Stock on 1-4-2007 100 units at Rs. 5 per unit.
Purchases
5-4-07 300 units at Rs. 6
8-4-07 500 units at Rs. 7
12-4-07 600 units at Rs. 8
Issues
6-4-07 250 units
10-4-07 400 units
14-4-07 500 units
Required :
(i) Calculate using FIFO and LIFO methods of pricing issues (you may assume a nil
opening and closing balance of stock on the production floor):
(a) the value of raw materials consumed during the period
(b) the value of stock of raw materials on 15-4-07.
(ii) Explain why the figures in (a) and (b) in part i of this question are different under the
two methods of pricing of raw material issues used. You need not draw up the
Stores Ledgers.

3
Material : Material Control and EOQ

4. (i) Explain the meaning of Material Control and give the objectives of material control
system.
(ii) A Company manufactures a special product which requires a component .Alpha.
The following particulars are collected for the year 2008 :
1 Annual demand of Alpha 8,000 units
2 Cost of placing an order Rs.200 per order
3 Cost per unit of Alpha Rs.400
4 Carrying cost % p.a. 20%
The company has been offered a quantity discount of 4 % on the purchase of Alpha
provided the order size is 4,000 components at a time.
Required :
(a) Compute the economic order quantity
(b) Advise whether the quantity discount offer can be accepted.

Labour : Allocation of Earnings to Jobs


5. Using the details given below, you are required to calculate the earnings of workers Rio
and Rayan and subsequently allocate these earnings to the three Jobs A, B and C.
Rio Rayan
(a) Basic Wages Rs. 100 Rs. 100
(b) Dearness Allowance 50% 55%
(c) Provident Fund (on basic wages) 8% 8%
(d) Employee’s State Insurance (on basic wages) 2% 2%
(e) Overtime 10 hrs. 
(f) Idle time and leave  16 hrs.
For your calculations, you may assume the following:
 Normal working hours for a month are 200 hours.
 Overtime is paid at double the normal wages plus dearness allowance.
 Employer’s contributions to State Insurance and Provident Fund are at equal
rate with the employee’s contributions.
 The month contains 25 working days and one paid holiday.

4
The two workers were employed on jobs A, B and C in the following proportions:
Job A B C
Worker A 80 60 60
Worker B 100 40 60
Overtime was done on job Y.
Labour : Incentive System & Bonus Plan
6. (i) What are the main factors that should be taken into account before introducing a
scheme of incentives?
(ii) Two workmen, .A. and .B., produce the same product using the same material.
Their normal wage rate is also the same. .A. is paid bonus according to the Rowan
system, while .B. is paid bonus according to the Halsey system. The time allowed to
make the product is 50 hours. .A. takes 30 hours while .B. takes 40 hours to
complete the product. The factory overhead rate is Rs.5 per man-hour actually
worked. The factory cost for the product for .A. is Rs.3,490 and for .B. it is Rs.3,600.
Required :
(a) Compute the normal rate of wages;
(b) Compute the cost of materials cost;
(c) Prepare a statement comparing the factory cost of the products as made by
the two workmen.

Overheads : Overhead Rats and Over/Under Absorption of Overheads


7. A cost centre in a factory furnishes the following working conditions:
Normal working week 40 hours
Number of machines 15
Normal weekly loss of hours on maintenance, etc. 4 hours per machine
Estimated annual overhead Rs. 1,55,520
Estimated direct wage rate Rs. 3 per hour
Number of weeks worked per year 48
Actual results in respect of a 4week period are:
Overhead incurred Rs. 15,000
Wages incurred Rs. 7,000
Machine-hours produced 2,200

5
You are required to:
(i) Calculate the overhead rate per machine-hour, and
(ii) Calculate the amount of under or over-absorption of both wages and overhead.

Overheads : Concept of Capacity


8. Explain the following concepts relating to capacity:
(i) Rated capacity
(ii) Practical capacity
(iii) Normal capacity
(iv) Idle capacity
Non : Integrated Accounting
9. As on 31st March, 2008, the following balances existed in a firm’s Cost Ledger :
Dr. Cr.
Rs. Rs.
Stores Ledger Control A/c 3,00,000
Work-in-Progress Control A/c 1,20,000
Finished Stock Ledger Control A/c 2,50,000
Manufacturing Overhead Control A/c 10,000
Cost Ledger Control A/c _______ 6,60,000
6,70,000 6,70,000
During the next three months the following items arose:
Rs.
Finished product (at cost) 2,10,000
Manufacturing overhead incurred 90,000
Raw materials purchased 1,23,000
Factory Wages 50,000
Indirect Labour 21,000
Cost of Sales 1,85,000
Material issued to production 1,27,000
Sales returned at Cost 5,000
Material returned to suppliers 3,000
Manufacturing overhead charged to production 77,000
You are required to pass Journal Entries and prepare Trial Balance.

6
Job Costing
10. ABC Ltd specialises in producing and packaging compact discs (CDs) for the music
recording industry. ABC Ltd uses a job order cost system. The following data
summarise the operations related to production for March, the first month of operations:
(a) Materials purchased on account, Rs. 15,500.
(b) Materials requisitioned and labour used:
Materials Factory Labour
Rs. Rs.
Job No. 100 2,650 1,770
Job No. 101 1,240 650
Job No. 102 980 420
Job No. 103 3,420 1,900
Job No. 104 1,000 500
Job No. 105 2,100 1,760
For general factory use 450 650
(c) Factory overhead costs incurred on account, Rs. 2,700.
(d) Depreciation of machinery, Rs. 1,750.
(e) Factory overhead is applied at a rate of 70% of direct labour cost.
(f) Jobs completed: Nos. 100, 101, 102, 104.
(g) Jobs 100, 101 and 102 were shipped, and customers were billed for Rs. 8,100, Rs.
3,800, and Rs. 3,500 respectively.
Instructions:
(i) Journalise the entries to record the transactions identified above.
(ii) Determine the account balances for Work in Process and Finished Goods.
(iii) Prepare a schedule of unfinished jobs to support the balance in the work in
process account.
(iv) Prepare a schedule of completed jobs on hand to support the balance in the
finished goods account.

Contract Costing
11. ABC Ltd is a construction company, which has undertaken three contracts. Information
for the previous year along with other details is provided to you below;

7
Contract A Contract B Contract C
(Rs.000). (Rs.000). (Rs.000)
Contract price 1,760 1,485 2,420
Balances brought forward at the beginning of the year:
Material on site 20 30
Written down value of plant and machinery 77 374
Wages accrued 5 10
Transactions during previous year:
Profit previously transferred to profit and loss a/c 35
Cost of work certified (cost of sales) 418 814
Transactions during current year:
Material delivered to site 88 220 396
Wages paid 45 100 220
Salaries and other cost 15 40 50
Written down value of plant issued to site 190 35
Head office expenses apportioned during the 10 20 50
year
Balances c/fwd at the end of the year:
Material on site 20
Written down value of plant and machinery 150 20 230
Wages accrued 5 10 15
Value of work certified at the end of the year 200 8 60 2100
Cost of work not certified at the end of the year 55

The agreed retention rate is 10% of the value of work certified by the contractee’s
architect. Contract C is scheduled to be handed over to the contractee in the near future.
It is estimated that Rs 3,05,000 shall be needed to be spent in addition to what has been
tabulated above to complete this particular contract. This amount includes an allowance
for plant depreciation, construction services and for contingencies.
You are required to prepare contract accounts for each of the three contracts and
recommend how much profit or loss should be taken up for the year.

Process Costing : Statement of Equivalent Production, Cost & Distribution Cost


12. Following details are related to the work done in Process ‘A’ XYZ Company during the
month of March, 2007 :

8
(Rs.)
Opening work-in progress (2,000 units)
Materials 80,000
Labour 15,000
Overheads 45,000
Materials introduced in Process ‘A’ (38,000 units) 14,80,000
Direct Labour 3,59,000
Overheads 10,77,000

Units scrapped : 3,000 units


Degree of completion :
Materials 100%
Labour and overheads 80%
Closing work-in progress : 2,000 units
Degree of completion :
Materials 100%
Labour and overheads 80%
Units finished and transferred to Process ‘B’ : 35,000 units
Normal Loss :
5% of total input including opening work-in-progress.
Scrapped units fetch Rs.20 per piece.
You are required to prepare :
(i) Statement of equivalent production
(ii) Statement of cost
(iii) Statement of distribution cost, and
(iv) Process ‘A’ Account, Normal and Abnormal Loss Accounts.

Process Costing : Process Cost and Finished Stock Account with Profit Element
13. Ivoke Co Ltd. produces product ABC which passes through two processes before it is
completed and transferred to finished stock. The following data relate to October 2008.
Processes Finished Stock
Particulars I II
Opening stock Rs.3,750 Rs.4,500 Rs.11,250

9
Direct materials 15,000 15,750 −
Direct wages 11,200 11,250 −
Factory overheads 10,500 4,500 −
Closing Stock 3,700 4,500 −
Inter-process profit included in opening 1,500 2250
stock
Output of Process I is transferred to Process II at 25% profit on the transfer price.
Output of Process II is transferred to finished stock at 20% profit on the transfer price
Stocks in process are valued at prime cost. Finished stock is valued at the price at which
it is received from Process II. Sales during the period is Rs.120,000
Required-Process Cost Accounts and Finished Stock Account showing the profit element
at each stage.

Operating Costing
14. Kirti Health Care Center consists of 20 beds. Unit is open for 300 days in a year. For 200
days, the unit has the full capacity of 20 patients per day and for balance 100 days, it has
an average 16 beds only occupied per day.
Following are the expenses: Rs.
Rent 15,000/- p.m.
Repair & Maintenance (fixed) 10,000/-
Food supplied to patients (variable) 72,000/-
Laundry charges (variable) 36,000/-
Medicines (variable) 60,000/-
Other expenses (fixed) 72,000/-
2 supervisor (each salary) 2,000/- p.m.
4 nurses (each salary) 2,000/- p.m.
2 ward boys (each salary) 1,000/- p.m.
The unit engaged expert doctors from outside to attend the patients and the fees were
paid to them on an average of Rs. 2,0000 per month. Fees of expert doctors were paid
on the basis of number of patients attended by them.
Required:
(i) Charge per day per patient to earn a profit of 100% on cost.
(ii) Number of patient days required by the unit to break even assuming above charge
per patient day.

10
Operating Costing
15. Mr. X owns a bus which runs according to the following schedule:
(a) Delhi to Chandigarh and back, the same day.
Distance covered: 150 kms. one way.
Number of days run each month : 8
Seating capacity occupied 90%.
(b) Delhi to Agra and back, the same day.
Distance covered: 120 kms. one way.
Number of days run each month : 10
Seating capacity occupied 85%.
(c) Delhi to Jaipur and back, the same day.
Distance covered: 270 kms. one way.
Number of days run each month : 6
Seating capacity occupied 100%.
(d) Following are the other details:
Cost of the bus Rs. 6,00,000
Salary of the Driver Rs. 2,800 p.m.
Salary of the Conductor Rs. 2,200 p.m.
Salary of the part-time Accountant Rs. 200 p.m.
Insurance of the bus Rs. 4,800 p.a.
Diesel consumption 4 kms per litre at Rs. 6 per litre
Road tax Rs. 1,500 p.a.
Lubricant oil Rs. 10 per 100 kms.
Permit fee Rs. 315 p.m.
Repairs and maintenance Rs. 1,000 p.m.
Depreciation of the bus @ 20% p.a.
Seating capacity of the bus 50 persons.
Passenger tax is 20% of the total takings. Calculate the bus fare to be charged from
each passenger to earn a profit of 30% on total takings. The fares are to be indicated
per passenger for the journeys:
(i) Delhi to Chandigarh
(ii) Delhi to Agra
(iii) Delhi to Jaipur.

11
Marginal Costing : Break Even Point, P/V Ratio and Margin of Safety
16. You are given the following data for the year 2007 of Rio Co. Ltd:
Variable cost 60,000 60%
Fixed cost 30,000 30%
Net profit 10,000 10%
Sales 1,00,000 100%
Find out
(i) Break-even point,
(ii) P/V ratio,
(iii) Margin of safety and,
(iv) Draw a break-even chart showing contribution and profit.

Marginal Costing : Income Statement According to Marginal & Absorption Costing


17 Aisha Automobiles assembles and sells motor vehicles. It uses an actual costing system,
in which unit cost are calculated on a monthly basis. Data relating to August and
September 2008 are
August September
Unit Data 2008 2008
Beginning Inventory 0 150
Production 500 400
Sales 350 520
Variable-cost data: Rs. Rs.
Manufacturing Costs per Unit Produced 10,000 10,000
Distribution cost per unit sold 3,000 3,000
Fixed-cost data:
Manufacturing Costs 22,00,000 22,00,000
Marketing Costs 4,00,000 4,00,000
The selling price per motor vehicle is Rs.24,000
Basic Data
August September
Production Units 500 400
Sales Unites 350 520
Opening inventory Units 0 150
Closing inventory units 150 30

12
Required
(i) Present income statement for Aisha Automobiles in August and September under :
(a) Marginal Costing
(b) Absorption Costing
(ii) Explain the differences between (a) and (b)

Standard Costing : Material and Labour Variance


18. The following standards have been set to manufacture a product:
Direct Material: Rs.
2 units of A @ Rs.4 per unit 8.00
3 units of B @ Rs.3 per unit 9.00
15 units of C @ Rs.1 per unit 15.00
32.00
Direct Labour: 3 hrs @ Rs.8 per hour 24.00
Total standard prime cost 56.00

The company manufactured and sold 6,000 units of the product during the year. Direct
material costs were as follows:
12,500 units of A at Rs.4.40 per unit
18,000 units of B at Rs.2.80 per unit
88,500 units of C at Rs.1.20 per unit
The company worked 17,500 direct labour hours during the year. For 2,500 of these
hours, the company paid at Rs.12 per hour while for the remaining, the wages were paid
at standard rate. Calculate materials price variance and usage variance and labour rate
and efficiency variances.

Standard Costing : Material Variance


19. Compute the missing data indicated by question mark from the following data given by
the Myanmar Ltd.
Particulars EXE PQP
Standard price/unit Rs.12 15
Actual price/unit Rs.15 20
Standard input (kg) 50 ?
Actual input (kg) ? 70

13
Material price variance ? ?
Material uses variance ? Rs.300 (A)
Material cost variance ? ?
Material mix variance for both the products together was Rs.90 (A)

Standard Costing : Overhead Variance


20. The following information was obtained from the records of a manufacturing unit using
standard costing system.
Standard Actual
Production 4,000 units 3,800 units
Working days 20 21
Fixed Overhead Rs. 40,000 Rs. 39,000
Variable Overhead 12,000 12,000
You are required to calculate the following overhead variance:
(i) Variable overhead variance
(ii) Fixed overhead variances
(a) Expenditure variances
(b) Volume variance

Budgetary Control : Functional and Flexible Budget


21. (i) Explain different kinds of functional budgets.
(ii) A department of AXY Company attains sales of Rs.6,00,000 at 80% of its normal
capacity. Its expenses are given below :
Rs. Selling Costs :
Office salaries 90,000 Salaries 8% of sales
General expenses 2% of sales Travelling expenses 2% of sales
Depreciation 7,500 Sales office 1% of sales
Rent and rates 8,750 General expenses 1% of sales
Distribution costs:
Wages 15,000
Rent 1% of sales
Other expenses 4% of sales
Draw up Flexible Administration, Selling and Distribution Costs Budget, operating at
90 per cent, 100 per cent and 110 per cent of normal capacity.

14
SUGGESTED ANSWERS/HINTS

Material : EOQ
2AB
1. (i) Economic Order Quantity (EOQ) =
CS
Where;
A = annual consumption,
B = buying cost per order,
C = cost per unit, and
S = storage and carrying cost

2 X18,250 units  Rs.50


=
Rs.36.50  20/100
1825000
= = 500 units
7.3

Labour : Halsey plan


(ii) Statement of Bonus, total earnings of labour and hourly earnings under Halsey and
Rowan Systems.
SH AH Time Basic Bonus Bonus Total Total Hourly Hourly
saved wages under under Earnings Earnings Earnings Earnings
(AH  Halsey Rowan under under under under
Rs.8) (B System system Halsey Rowan Halsey Rowan
 Rs.8)  50  B  System System System System
 100  C  8  A  C  8 D+E D+F G/B H/B
   

A B C= D E F G H I J
hours hours (A-B) Rs. Rs. Rs. Rs. Rs. Rs. Rs.
hours
8 8 - 64 - - 64 64 8.00 8.00
8 7 1 56 4 7 60 63 8.57 9.00
8 6 2 48 8 12 56 60 9.33 10.00
8 5 3 40 12 15 52 55 10.40 11.00
8 4 4 32 16 16 48 48 12.00 12.00
8 3 5 24 20 15 44 39 14.67 13.00
8 2 6 16 24 12 40 28 20.00 14.00
8 1 7 8 28 7 36 15 36.00 15.00

15
Standard Costing
(iii) Activity ratio = Capacity ratio  Efficiency ratio
Activity ratio 104
 Efficiency ratio =   108.33%
Capacity ratio 96

Material : Inventory Turnover Ratio


(iv)
Rs.
Cost of raw material consumed
Opening stock of raw material on 1-4-2007 90,000
Add: Purchases during the year 2,70,000
3,60,000
Less:Closing stock of raw material 1,10,000
Cost of raw material consumed 2,50,000

Cost of stock of raw material consumed


(i) Inventory turnover ratio =
Average stock of raw material
= Rs.2,50,000  {(90,000 + 1,10,000)/2} = 2.5
(ii) Average number of days for which the average inventory is held
365 days
Or Inventory turnover period =  146 days
2.5
Operating Costing
(v) Absolute tonnes - kms. = tonnes × Kms.
= (20 tonnes × 80 kms.) + (12 tonnes × 120 kms.) + (16 × 160 kms.)
= 5,600 tonnes - kms.
Commercial tonnes-kms.
= Average load × Total kms. Covered.
= (20 + 12 + 16)/3 tonnes × 360 kms.
= 5,760 tonnes-kms.

16
Labour : Rate per hour and wages
(vi)
Standard time 60 hours
Actual time taken 50 hours
(presumed)
Time saved 10 hours
Level of efficiency S tan dard Time
= 100
Actual Time
60
= 100  120%
50
Normal Rate Rs.5.00
Add: Extra benefit (i) For 100%
efficiency 10% extra on
Re.050
hourly rate
(ii) For more than
100% efficiency 1%
increase for every
additional 1%
efficiency- 20% of Rs.5
Adjusted Hourly Rate. Rs. 1.00
Rs. 6.50
Wage payable to the worker: 50 × Rs.6.50 = Rs.325.
Marginal Costing : Margin of Safety
C 10  8 2
(vii) Profit/Volume Ratio =  100   100   100  20%
S 10 10
Pr ofit 30,000
Margin of safety =   Rs.1,50,000
P / V ratio 20%

Application of Marginal Costing and Flexible Budget


(viii) P/V ratio = 100 – 70 = 30%
F 90,000
BE point =  Rs.3,00,000
P / V ratio 30%
3,00,000
When BE point at 60% is Rs.3,00,000, capacity sales =  Rs.5,00,000
60%
Sales at 75% capacity = 5,00,000  75% = Rs.3,75,000
Profit = (Sales  30%) – 90,000 = Rs.22,500

17
Marginal Costing : Break – Even point
(ix)
Sales Profit/Loss(-)
Rs. Rs.
8,000 units 1,20,000 (-)40,000
20,000 units 3,00,000 80,000
Difference (Change in Sales/Profit) 1,80,000 1,20,000
P/V ratio = Difference in profit 1,20,000 2
 
Difference in sales 1,80,000 3
Fixed cost = Sales  P/V ratio - Profit
= (3,00,000  2/3) – 80,000 = Rs.1,20,000
BEP(Rs.) = F 1,20,000
  Rs.1,80,000
P / V ratio 2/3
BEP (in units) = Sales in Rs.  Selling price
= 1,80,000  Rs.15 = 12,000 units.

Process Costing
(x) Output in a process is input minus loss in process. Suppose input in process A =
100 Kg.
Output in Process A = 100 – 25% = 75 Kg
Output in Process B = 75 kg – 20% = 60 Kg
Output in Process C = 60 kg –20% = 48 Kg
Output in Process D = 400 kg–16 ⅔ % = 40 Kg
When Output in Process D is 40,000 kg, then input in Process A will be
100
 40,000 kg. 
40
= 1,00,000 kg.
Thus cost of materials introduced in Process I = 1,00,000 kg. @ Rs. 5 =
Rs. 5,00,000.

18
Introduction to Cost Accounting: Explanation of Various Terms
2. (i) Direct costs – Costs that are related to the cost object and can be traced in an
economically feasible way.
(ii) Marginal Cost - The amount at any given volume of output by which aggregate
costs are changed if the volume of output is increased or decreased by one unit.
(iii) Responsibility Centre - It is defined as an activity centre of a business
organisation entrusted with a special task. Under modern budgeting and control,
financial executives tend to develop responsibility centres for the purpose of control.
Responsibility centres can broadly be classified into three categories. They are Cost
Centres ; Profit Centres and Investment Centres.
(iv) Cost absorption - It is defined as the process of absorbing all indirect costs
allocated to or apportioned over a particular cost centre or production department
by the units produced. Hence, while allocating, the relevant cost objects would be
the concerned cost centre or the concerned department, while, the process of
absorption would consider the units produced as the relevant cost object. For
example, the overhead costs of a lathe centre may be absorbed by using a rate per
lathe hour. Cost absorption can take place only after cost allocation. In other words,
the overhead costs are either allocated or apportioned over different cost centres
and afterwards they are absorbed on equitable basis by the output of the same cost
centres.
(v) Imputed costs - These costs are notional costs which do not involve any cash
outlay. Interest on capital, the payment for which is not actually made, is an
example of imputed cost. These costs are similar to opportunity costs.
(vi) Out-of-pocket cost - It is that portion of total cost, which involves cash outflow.
This cost concept is a short-run concept and is used in decisions relating to fixation
of selling price in recession, make or buy, etc. Out–of–pocket costs can be avoided
or saved if a particular proposal under consideration is not accepted.
(vii) Shut down costs - Those costs, which continue to be, incurred even when a plant
is temporarily shutdown, e.g. rent, rates, depreciation, etc. These costs cannot be
eliminated with the closure of the plant. In other words, all fixed costs, which cannot
be avoided during the temporary closure of a plant, will be known as shut down
costs.
(viii) Sunk costs - Historical costs incurred in the past are known as sunk costs. They
play no role in decision making in the current period. For example, in the case of a
decision relating to the replacement of a machine, the written down value of the
existing machine is a sunk cost and therefore, not considered.
(ix) Discretionary costs – Such costs are not tied to a clear cause and effect
relationship between inputs and outputs. They usually arise from periodic decisions

19
regarding the maximum outlay to be incurred. Examples include advertising, public
relations, executive training etc.
(x) Engineered costs - These are costs that result specifically from a clear cause and
effect relationship between inputs and outputs. The relationship is usually
personally observable. Examples of inputs are direct material costs, direct labour
costs etc. Examples of output are cars, computers etc.

Material : FIFO/LIFO
3. (i) (a) Value of Material ASH consumed during the period 1-4-06 to 15-4-07 by
using FIFO method
Date Description Units Qty Rate Amount
Rs. Rs.
1-4-07 Opening balance 100 5 500
5-4-07 Purchased 300 6 1,800
6-4-07 Issued 100 5
150  1,400
6
8-4-07 Purchased 500 7 3,500
10-4-07 Issued 150 6
250  2,650
7
12-4-07 Purchased 600 8 4,800
14-4-07 Issued 250 7
250  3,750
8
15-4-07 Balance 350 8 2,800
Total value of material ASH consumed during the period under FIFO method
comes to (Rs. 1,400 + Rs. 2,650 + Rs. 3,750) Rs. 7,800 and balance on 15-4-
07 is of Rs. 2,800.
Value of Material ASH consumed during the period
1-4-06 to 15-4-06 by using LIFO method

Date Description Qty. Rate Amount

Units Rs. Rs.

1-4-07 Opening balance 100 5 500

5-4-07 Purchased 300 6 1,800

6-4-07 Issued 250 6 1,500

20
8-4-07 Purchased 500 7 3,500

10-4-07 Issued 400 7 2,800

12-4-07 Purchased 600 8 4,800

14-4-07 Issued 500 8 4,000

15-4-07 Balance 350 — 2,300*

Total value of material ASH issued under LIFO method comes to (Rs. 1,500 +
Rs. 2,800 + Rs. 4,000) Rs. 8,300.
*The balance 350 units on 15-4-07 of Rs. 2,300, relates to opening balance on
1-4-07 and purchases made on 5-4-07, 8-4-07 and 12-4-07. (100 units @ Rs.
5, 50 units @ Rs. 6, 100 units @ Rs. 7 and 100 units @ Rs. 8).
(b) As shown in (a) above, the value of stock of materials on 15-4-07:
Under FIFO method Rs. 2,800
Under LIFO method Rs. 2,300
(ii) Total value of material ASH issued to production under FIFO and LIFO methods
comes to Rs. 7,800 and Rs. 8,300 respectively. The value of closing stock of
material ASH on 15-4-07 under FIFO and LIFO methods comes to Rs. 2,800 and
Rs. 2,300 respectively.
The reasons for the difference of Rs. 500 (Rs. 8,300 – Rs. 7,800) as shown by the
following table in the value of material ASH, issued to production under FIFO and
LIFO are as follows:

Date Quantity Value Total Value Total


Issued FIFO LIFO
(Units) Rs. Rs. Rs. Rs.
6-4-07 250 1,400 1,500
10-4-07 400 2,650 2,800
14-4-07 500 3,750 7,800 4,000 8,300
1. On 6-4-07, 250 units were issued to production. Under FIFO their value comes
to Rs. 1,400 (100 units × Rs. 5 + 150 units × Rs. 6) and under LIFO Rs. 1,500
(250 × Rs. 6). Hence, Rs. 100 was more charged to production under LIFO.
2. On 10-4-07, 400 units were issued to production. Under FIFO their value
comes to Rs. 2,650 (150 × Rs. 6 + 250 × Rs. 7) and under LIFO Rs. 2,800
(400 × Rs. 7). Hence, Rs. 150 was more charged to production under LIFO.

21
3. On 14-4-07, 500 units were issued to production. Under FIFO their value
comes to Rs. 3,750 (250 × Rs. 7 + 250 × Rs. 8) and under LIFO Rs. 4,000
(500 × Rs. 8). Hence, Rs. 250 was more charged to production under LIFO.
Thus the total excess amount charged to production under LIFO comes to Rs. 500.
The reasons for the difference of Rs. 500 (Rs. 2,800 – Rs. 2,300) in the value of
350 units of Closing Stock of material ASH under FIFO and LIFO are as follows :
1. In the case of FIFO, all the 350 units of the closing stock belongs to the
purchase of material made on 12-4-07, whereas under LIFO these units were
from opening balance and purchases made on 5-4-07, 8-4-07 and 12-4-07.
2. Due to different purchase price paid by the concern on different days of
purchase, the value of closing stock differed under FIFO and LIFO. Under
FIFO 350 units of closing stock were valued @ Rs. 8 p.u. Whereas under LIFO
first 100 units were valued @ Rs. 5 p.u., next 50 units @ Rs. 6 p.u., next 100
units @ Rs. 7 p.u. and last 100 units @ Rs. 8 p.u.
Thus under FIFO, the value of closing stock increased by Rs. 500.

Material : Material Control and EOQ


4. (i) Material Control : The publication of the Institute of Cost and Management
Accountants on Budgetary Control defines it as .the function of ensuring that
sufficient goods are retained in stock to meet all requirements without carrying
unnecessarily large stocks. When the functions of indexing buying, receiving,
inspection, storing and paying of the goods are separated it is essential that these
should be properly co-ordinated so as to achieve the advantages of specialisation.
Objectives of Material Control System: The objectives of a system of material
control are the following :
(a) Ensuring that no activity, particularly production, suffers from interruption for
want of materials and stores-it should be noted that this requires constant
availability of every item that may be needed howsoever small its cost may be.
Lubricating oil may cost much less than the main raw material but, from the
point of view of uninterrupted production, both have equal importance.
(b) Seeing to it that all the materials and stores are acquired at the lowest possible
price considering the quality that is required and considering other relevant
factors like reliability in respect of delivery, etc.
(c) Minimisation of the total cost involved, both for acquiring stocks (apart from the
price paid to the supplier) and for holding them.
(d) Avoidance of unnecessary losses and wastages that may arise from
deterioration in quality due to defective or long storage or from obsolescence.

22
It may be noted that losses and wastages in the process of manufacture,
concern the production department.
(e) Maintenance of proper records to ensure that reliable information is available
for all items of materials and stores that not only helps in detecting losses and
pilferages but also facilitates proper production planning.
(ii) (a) Calculation of Economic Order Quantity
2AB
EOQ =
CS
Where;
A = annual consumption,
B = buying cost per order,
C = cost per unit, and
S = storage and carrying cost
2  8,000 units  Rs.200
= 200 units
Rs.400  20/100
(b) Evaluation of Profitability of Different Options of Order Quantity
(1) When EOQ is ordered (Rs.)
Purchase Cost (8,000 units  Rs 400) 32,00,000
Ordering Cost [(8,000 units/200 units)  Rs.200] 8,000
Carrying Cost (200 units  Rs.400  ½  20/100) 8,000
Total Cost 32,16,000

(2) When Quantity Discount is accepted (Rs.)


Purchase Cost (8,000 units  Rs.384*) 30,72,000
Ordering Cost [(8,000 units/4000 units)  Rs.200] 400
Carrying Cost (4000 units  Rs.384  ½  20/100) 1,53,600
Total Cost 32,26,000
Advise – The total cost of inventory is lower if EOQ is adopted. Hence,
the company is advised not to accept the quantity discount.
* 400 – (4% of 400) = 384

23
Labour : Allocation of Earnings to Jobs
5. Computation of wages payable
Worker Rio Worker Rayan
Rs. Rs. Rs.
Basic Wages 100 100
Dearness Allowance 50 55
(1) Overtime 10 hours @ Rs. 1.50 15
Gross Wages 165 155
Less: Deductions
Employees’ contribution to provident Fund (8)
(8% on basic wages)
Employees’ contribution to E.S.I. (2) (10) (10)
(2% on basic wages)
Net wages Payable 155 145

Allocation of Labour Cost to Jobs


Worker Rio Worker Rayan
Rs. Rs.
Gross Wages (excluding overtime wages 150 155
Employer’s contribution to P.F. and E.S.I. 10 10
160 165
Normal working hours per month 200 hrs 200 hrs
Labour cost per hour 160 165
 Re.0.80  0.825
200 200

Total Job A Job B Job C


Rs. Rs. Rs. Rs.
Worker Rio Ordinary wages 160.00 64.00 48.00 48.00
Overtime wages 15.00 15.00
Worker Rayan Ordinary wages 165.00 82.50 33.00 49.50
340.00 146.50 96.00 97.50

24
Working Notes:
Calculation of overtime wages
Rs.
Basic wages per month 100
Dearness allowance 50
150
Normal working hours per month 200 hours

Rs. 150
Normal rate per hour =  Re. 0.75
200
Therefore, overtime rate is Rs 1.50 because overtime is paid at double the normal rate.
Overtime wages for 10 hours @ Rs 1.50 = Rs 15.

Labour : Incentive System & Bonus Plan


6. (i) The main factors that should be taken into account before introducing a scheme of
incentives are stated below :
(a) The need for producing goods of high quality or those having very good
workmanship or finish and the manner this can be ensured. Only if a system of
quality control can be relied upon to maintain the quality of goods of the
standard required, an incentive scheme should be introduced; otherwise,
workers should be paid on time basis.
(b) The need to maximise production—thus required incentives to be given to
workers. But sometimes workmanship is more important than quantity of
output; in such cases, incentive schemes of wage payment are not suitable.
(c) Where the quantity of work done cannot be measured precisely, incentive
schemes cannot be offered.
(d) The role of management and workers in achieving greater efficiency, if it is
unnecessary for the management to constantly plan work, for example, when
the work is repetitive, workers should be offered good incentives to achieve
high efficiency; but in case management is constantly required to plan the
work, as in the case of job work, the management should share the fruits of
extra efficiency achieved. This factor determines the choice of a particular
incentive scheme.
(e) Whether the quantity of output is within the control of the worker and if so, to
what extent. Sometimes, as in the case of chain assembly work the output is
not dependent on the effort put in by workers; incentive schemes in such cases
are not suitable.

25
(f) The exactitude with which standards of performance can be laid down. Fixation
of standard is necessary for the introduction of a scheme of incentives. When
this requires heavy expenditure, incentive schemes may be rather costly.
(g) The effect of an incentive scheme for one set of workers on other workers. If
for instance, an incentive scheme makes it possible for unskilled workers to
earn high wages, the wage rates for skilled workers must also be raised (if
they are paid on time basis) to avoid dissatisfaction among them. In that event,
the incentive scheme may raise labour cost instead of lowering it.
(h) The system of wage payment prevailing in other areas and industries or similar
occupations. If possible, there should be uniformity.
(i) The attitude of labour and trade unions towards incentive schemes. Workers
usually like to have a certain guaranteed time-basis wage but also like to earn
extra through an incentive scheme.
(ii) Step 1 : Let X be the cost of material and Y be the normal rate of wages per hour.
Step 2 : Factory Cost of Workman ‘A’
Rs.
A. Material Cost X
B. Wages 30 Y
AH 12 Y
C. Bonus =  (SH - AH)  R
SH
30
Bonus =  (50 - 30)  Y
50
D. Overheads (30  Rs.5) 150
E. Factory Cost X + 30Y + 12 Y + Rs.150
or, = X + 42 Y = Rs.3,490 (Given) – Rs.150 = Rs.3,340 … Equation (I)
Step 3 : Factory Cost of Workman ‘B’
Rs.
A. Material Cost X
B. Wages 40 Y
C. Bonus = 50% of (SH - AH)  R 5Y
= 50% of (50 - 40)  R
D. Overheads (40  Rs.5) 200
E. Factory Cost X + 40Y + 5 Y + Rs.200
or, = X + 45 Y = Rs.3,600 (Given) – Rs.200 = Rs.3,400 … Equation (II)

26
Step 4 : Subtracting Eq. (I) from Eq. (II)
3Y = Rs.60
Y = Rs.60/3 = Rs.20 per hour.
(a) The normal rate of wages : Rs.20 per hour
(b) The cost of material : X + 45  Rs.20 = Rs.3,400
X = Rs.3,400 – Rs.900 = Rs.2,500
(c) Comparative Statement of the Factory Cost of the product made by the two
workmen.
‘A’ ‘B’
Rs. Rs.
Material cost 2,500 2,500
Direct Wages (30  Rs.20) 600 (40  Rs.20) 800
Bonus (12  Rs.20) 240 (5  Rs.20) 100
Factory Overhead 150 200
Factory Cost 3,490 3,600
Overheads : Overhead Rats and Over/Under Absorption of Overheads
7. (i) Annual overhead – Rs. 1,55,520
Annual workings hours (Normal) = Number of machines  No. of weeks p.a. 
Effective weekly hours
= 15  48  36 = 25,920
Overhead rate per machine hour = Rs. 1,55,520 ÷ 25,920 = Rs. 6.00
(ii) Overhead incurred = Rs. 15,000
Overhead absorbed = Rs. 13,200 (i.e., 2,200  Rs. 6)
Under-absorbed overhead = 15,000 – 31,200 = Rs. 1,800
Wages incurred = Rs. 7,000
Wages absorbed = Rs. 7,200
(i.e., 40 hours  15 machines  Rs. 3  4 weeks)
Wages over-absorbed = 7,200 – 7,000 = Rs. 200.

Overheads : Concept of Capacity


8. (i) Rated capacity : It refers to the capacity of a machine or a plant as indicated by its
manufacturer. In fact this capacity is the maximum possible productive capacity of a
plant. It is also known as installed capacity of a plant. Due to the loss of operating

27
time of a plant it is difficult to achieve this rated capacity. In other words, it is only a
theoretical capacity and is therefore, seldom achieved.
(ii) Practical capacity : It is defined as actually utilised capacity of a plant. It is also
known as operating capacity. This capacity takes into account loss of time due to
repairs, maintenance, minor breakdown, idle time, set up time, normal delays,
Sundays and holidays, stock taking etc. Generally, practical capacity is taken
between 80 to 90% of the rated capacity. It is also used as a base for determining
overhead rates. Practical capacity is also called net capacity or available capacity.
(iii) Normal capacity : It is the capacity of a plant which is expected to be utilised over a
long period based on sales expectancy. The determination of this capacity
considers the average utilisation of plant capacity during one full business cycle
which may extend over 2 to 3 years. It is also known as average capacity and is
used to compute overhead recovery rate.
(iv) Idle capacity : It is that part of the capacity of a plant, machine or equipment which
cannot be effectively utilised in production. In other words, it is the difference
between the practical or normal capacity and capacity utilisation based on expected
sales. For example, if the practical capacity of production of a machine is to the
tune of 10,000 units in a month, but is used only to produce 8,000 units, because of
market demand of the product, then in such a case, 2,000 units will be treated as
idle capacity of the machine.
Non : Integrated Accounting
9. Journal entries are as follows:
Dr. Cr.
Rs. Rs.
1. Finished stock ledger Control A/c Dr. 2,10,000
To Work-in-Progress Control A/c 2,10,000
2. Manufacturing Overhead Control A/c Dr. 90,000
To Cost Ledger Control A/c 90,000
3. Stores Ledger Control A/c Dr. 1,23,000
To Cost Ledger Control A/c 1,23,000
4. (i) Wage Control A/c Dr. 71,000
To Cost Ledger Control A/c 71,000
(ii) Work-in-progress Control A/c Dr. 50,000
To Wage Control A/c 50,000

28
(iii) Manufacturing Overhead Control A/c Dr. 21,000
To Wage Control A/c 21,000
5. Cost of Sales A/c Dr. 1,85,000
To Finished Stock Ledger A/c 1,85,000
6. Work-in-Progress Control A/c Dr. 1,27,000
To Stores Ledger Control A/c 1,27,000
7. Finished Stock Ledger Control A/c Dr. 5,000
To Cost of Sales A/c 5,000
8. Cost Ledger Control A/c Dr. 3,000
To Stores Ledger Control A/c 3,000
9. Work-in-Progress Control A/c Dr. 77,000
To Manufacturing Overhead Control A/c 77,000

Ledger Posting

Cost Ledger Control Account

Rs. Rs.
To Stores Ledger Control A/c (return) 3,000 By Balance b/d 6,60,000

”Balance c/d 9,41,000 ” Manufacturing Overhead


Control A/c 90,000
” Stores Ledger Control A/c 1,23,000
_______ ” Wage Control A/c 71,000
9,44,000 9,44,000

Stores Ledger Control Account


Rs. Rs.
To Balance b/d 3,00,000 By Work-in-Progress
” Cost Ledger Control A/c 1,23,000 ” Control A/c 1,27,000
” Cost Ledger Control A/c 3,000
_______ ” Balance c/d 2,93,000
4,23,000 4,23,000

29
Work-in-Progress Control Account
Rs. Rs.
To Balance b/d 1,20,000 By Finished Stock
” Wage Control A/c 50,000 Ledger Control A/c 2,10,000
” Stores Ledger Control A/c 1,27,000 ” Balance c/d 1,64,000
Manufacturing Overhead
Control A/c 77,000 _______
3,74,000 3,74,000

Finished Stock Ledger Control Account


Rs. Rs.
To Balance b/d 2,50000 By Cost of Sales A/c 1,85,000
Work-in-Progress ” Balance c/d 2,80,000
Control A/c 2,10,000
” Cost of Sales A/c(return at cost) 5,000 _______
4,65,000 4,65,000

Manufacturing Overhead Control Account


Rs. Rs.
To Cost Ledger Control A/c 90,000 By Balance b/d 10,000
” Wage Control A/c 21,000 ” Work-in-ProgressControl A/c 77,000
” Balance c/d (under recovered) 24,000
1,11,000 1,11,000

Wage Control Account


Rs. Rs.
To Cost Ledger Control A/c 71,000 By Work-in-Progress Control A/c 50,000
” Manufacturing Overhead
Control A/c 21,000
71,000 71,000

30
Cost of Sales Account
Rs. Rs.
To Finished Stock Ledger Control A/c1,85,000 By Finished Stock Ledger
Control A/c (Return) 5,000
” Balance c/d 1,80,000
1,85,000 1,85,000
Trial Balance
Dr. Cr.
Rs. Rs.
Stores Ledger Control A/c 2,93,000
Work-in-Progress Control A/c 1,64,000
Finished Stock Ledger Control A/c 2,80,000
Manufacturing Overhead Control A/c 24,000
Cost of Sales A/c 1,80,000
Cost Ledger Control A/c _______ 9,41,000
9,41,000 9,41,000
Job Costing
10. (i) Journal entries are as follows
(a) Materials 15,500
To Accounts Payable 15,500
(b) Work in process 11,390
To Materials 11,390
Work in Process 7,000
To Wages payable 7,000
Factory Overhead 1,100
To Materials 450
To Wages Payable 650
(c) Factory Overhead 2,700
To Accounts Payable 2,700
(d) Factory Overhead 1,750
To Accumulated Depreciation  Machinery 1,750

31
(e) Work in Process 4,900
To Factory Overhead (70% of Rs. 7,000) 4,900
(f) Finished Goods 11,548
To Work in Process 11,548
g. Accounts receivable 15,400
To Sales 15,400
Cost of Goods sold 9,698
To Finished Goods 9,698

(ii) Computation of the cost of jobs finished:


Job Direct Direct Factory Total
Materials Labour Overhead
Rs. Rs. Rs. Rs.
Job No. 100 2,650 1,770 1,239 5,659
Job No. 101 1,240 650 455 2,345
Job No. 102 980 420 294 1,694
Job No. 104 1,000 500 350 1,850
11,548
Cost of jobs sold computation:
Rs.
Job No. 100 5,659
Job No. 101 2,345
Job No. 102 1,694
9,698
Work in Process : Rs. 11,742 (Rs. 11,390 + Rs. 7,000 + Rs. 4,900 – Rs. 11,548)
Finished Goods: Rs. 1,850 (Rs. 11,548 – Rs. 9,698)

(iii) Schedule of unfinished jobs


Job Direct Direct Factory Total
Materials Labour Overhead
Rs. Rs. Rs. Rs.
Job No. 103 3,420 1,900 1,330 6,650

32
Job No. 105 2,100 1,760 1,232 5,092
Balance of work in process, March 31 11,742
(iv) Schedule of Completed Jobs
Rs.
Job No. 104:
Direct Materials 1,000
Direct Labour 500
Factory overhead 350
Balance of Finished Goods, March 31 1,850
Contract Costing
11. Contract Accounts (in Rs 000)
A B C A B C
Material on site 20 30 Wages accrued 5 10
b/fwd b/fwd
Plant on site b/fwd 77 374 Material on site 20
c/fwd
Material control a/c 88 220 396 Plant on site c/fwd 150 20 230
Wages control a/c 45 100 220 Cost of work not 55
certified c/fwd
Salaries 15 40 50 Cost of sales – 183 497 840
current period
(balance) c/fwd
Plant control a/c 190 35
Apportionment of 10 20 50
HO expenses
Wages accrued 5 10 15
c/fwd
353 522 1,135 353 522 1,135
Cost of sales b/fwd 183 497 840 Attributable sales 183 442 1,122
revenue (current
period)*

33
Profit taken this 282 Loss taken 55
period
183 497 1,122 183 497 1,122
Cost of work not certified 55 Wages accrued 5 10 15
b/fwd b/fwd
Material on site 20
b/fwd
Plant on site b/fwd 150 20 230
* Profit taken plus cost of sales for the current period or cost of sales less loss to date
Note
 Profit/loss on the three contracts are calculated by deducting the cost of sales (both
previous years and current year) from the value of work certified
(Rs 000)
Contract A 17 (Rs 200 – Rs 183)
Contract B (55) (Rs 860 – Rs 915)
Contract C 446 (Rs 2,100 – Rs 1,654)
Recommendation
Computation of profit taken for Contract C is as follows
(Rs000)
Cost of work certified(cost of sales to date = 814 + 840) 1,654
Cost of work not certified 55
Estimated costs to complete 305
Estimated cost of contract 2,014
Contract price 2,420
Anticipated profit 406
(0.90  Rs2,100)
Profit taken =  Rs 406 less profit previously transferred
Rs2,420
= Rs 3,17,000 – Rs 35,000 = Rs 2,82,000
 No profit has been taken for Contract A as it is in very early stages of completion
 Prudence concept has been utilized for Contract B. All loss has been taken.

34
Process Costing : Statement of Equivalent Production, Cost & Distribution Cost
12. (i) Statement of Equivalent Production
Input Details Units Output Details Units Equivalent Production
Material Labour &
O.H.
Units % Units %
Opening WIP 2,000 Completed and transferred 35,000 35,000 100 35,000 100
to process B
Units 38,000 Normal Loss (5% of 40,000) 2,000    
introduced
Abnormal loss 1,000 1,000 100 800 80
_____ Closing WIP 2,000 2,000 100 1,600 80
40,000 40,000 38,000 37,400

(ii) Statement of Cost


Details Cost at the Cost Total Cost Equivalent Cost per
beginning added (Rs.) Production unit (Rs.)
of the (Rs.) (units)
process
(Rs.)
Material 80,000 14,80,000 15,60,000
Less : Value of Normal Scrap 40,000
15,20.000 38,000 40
Labour 15,000 3,59,000 3,74,000 37,400 10
Overheads 45,000 10,77,000 11,22,000 37,400 30
80
(iii) Statement of Distribution of Cost Rs.)
i) Completed and transferred to Process B (35,000 units @ Rs.80) 28,00,000
ii) Abnormal Loss – 1,000 units
Materials (1,000 units @ Rs.40) 40,000
Labour and overheads (800 units @ Rs.40) 32,000 72,000
iii) Closing W.I.P. – 2,000 units
Materials (2,000 units @ Rs.40) 80,000
Labour and overheads (1,600 units @ Rs.40) 64,000 1,44,000

35
(iv) Process A A/c
Particulars Units Rs. Particulars Units Rs.
To Balance:
Opening W.I.P. 2,000 1,40,000 By Normal Loss 2,000 40,000
Materials – Rs.80,000 (@Rs.20 per paise)
Labour – Rs.15,000 By Abnormal loss 1,000 72,000
Overheads – Rs.45,000 By Process B A/c 35,000 28,00,000
To Materials introduced 38,000 14,80,000 (transferred to the next Process)
To Direct Labour 3,59,000 Balance c/d (Closing WIP) 2,000 1,44,000
To Overheads _____ 10,77,000 By 40,000 30,56,000
40,000 30,56,000

Normal Loss A/c


Particulars Units Rs. Particulars Units Rs.
To Process A A/c 2,000 40,000 By Cost Ledger Control A/c 2,000 40,000

Abnormal Loss A/c


Particulars Units Rs. Particulars Units Rs.
To Process A A/c 1,000 72,000 By Cost Ledger Control A/c 1,000 20,000
______ By Costing Profit / Loss A/c 52,000
72,000 72,000

Process Costing : Process Cost and Finished Stock Account with Profit Element
13. Process I Account
Particulars Cost Profit Total Particulars Cost Profit Total
Rs. Rs. Rs. Rs. Rs. Rs.
To Opening Stock 3,750 − 3,750 By Process II 3,675 12,250 49,000
To Direct Material 15,000 − 15,000
To Direct Wages 11,200 − 11,200
29,950 − 29,950
Less: Closing Stock 3,700 − 3,700
26,250 − 26,250
To Factory Overheads 10,500 − 10,500
36,750 − 36,750
To Profit 12,250 _____ 12,250 ______ ______ ______
36,750 12,250 49,000 36,750 12,250 49,000

36
Process II Account
Particulars Cost Profit Total Particulars Cost Profit Total
Rs. Rs. Rs. Rs. Rs. Rs.
To Opening Stock 3,000 1,500 4,500 By Process II 67,519 33,106 10,0625
To Process I 36,750 12,250 49,000
To Direct Material 15,750 − 15,750
To Direct Wages 11,250 − 11,250
Less: Closing Stock 66,750 13,750 80,500
3,731 769 4,500
63,019 12,981 76,000
To Factory Overheads 4,500 − 4,500
67,519 12,981 80,500
To Profit − 20,125 20,125 ______ ______ _______
67,519 33,106 10,0625 67,519 33,106 1,00,625
Finished Stock Account
Particulars Cost Profit Total Particulars Cost Profit Total
Rs. Rs. Rs. Rs. Rs. Rs.
To Opening Stock 9,000 2,250 11,250 By Sales 68,824 51,176 1,20,000
To Process II 67,519 33,106 1,00,625
76,519 35,356 1,11,875
Less: Closing Stock 7,695 3,555 11,250
68,824 31,801 1,00,625
To Profit − 19,375 19,375 ______ ______ _______
68,824 51,176 1,20,000 68,824 51,176 1,20,000
Working Notes:
1. Reserve for unrealised profit in closing stock.
Process I. Nil
Process II
Cost 66,750
Cost of stock   closing stock   4,500  3,731
Total 80,500
Profit = Total – Cost = 4,500 – 3,750 = Rs.750
Finished Stock
Cost 76,519
Cost of stock   closing stock   11,250  7,695
Total 1,11,875
Profit = Total – Cost = 11,250 – 7,695 = Rs.3,555

37
2. Profit for the month
Rs. Rs.
Process I − − − − 12,250
Process II − − − 20,125
Add: Unrealised profit in opening stock − − − 1,500
21,625
Less: Unrealised Profit in Closing stock − − − 769 20,856
Finished Stock − − − 19,375
Add: Unrealised profit in opening stock − − − 2,250
21,625
Less: Unrealised profit in closing stock − − − 3,555 18,070
Total Profit 51,176

Operating Costing
14. (i) Statement showing cost per patient day
A. variable costs
i. Foods 74,000
ii. Laundry charges 36,000
iii Medicines 60,000
iv. Doctor fees 240000 4,10,000
B. Fixed costs
i. Repair & Maintenance 10,000
ii. Rent 1,80,000
iii. Other fixed expenses 72,000
iv. Supervisors’ salary (2x12x2000) 48,000
v. Nurses’ salary (4x12x2000) 96,000
vi. Ward boys’ salary (2x12x1000) 24,000 4,30,000
Total costs (A+B) 840,000
Add: Profit (100% an total cost) 840,000
C. Total Revenue Required 16,80,000
D. No. of patient days (Working Note1) 5600
Charge per day per patient (C/D) 300

38
Fixed costs
(ii) Break even point =
Contribution per patient day
Revenue per patient day = Rs. 300
Rs.410000
Variable cost per patient day = = Rs. 73.21
5600
Contribution per patient day = Rs.(300 – 73.21) = Rs. 226.79
430000
B.E.P. = = 1896 patient days
226.79

Operating Costing
15. Total kms. covered per month
Bus route Km. per trip Trips per day Days per month Km. per month
Delhi to Chandigarh 150 2 8 2,400
Delhi to Agra 120 2 10 2,400
Delhi to Jaipur 270 2 6 3,240
8,040
Passenger kms. per month
Total seats Capacity Km. per Passenger
available per utilised trip Km. per
month month
(100% capacity) % Seats
Delhi to Chandigarh & Back 800* 90 720 150 1,08,000
Delhi to Agra & Back 1,000@ 85 850 120 1,02,000
Delhi to Jaipur & Back 600# 100 600 270 1,62,000
Total 3,72,000
* 50 seats  2 trips  8 days = 800 seats
@ 50 seats  2 trips  10 days = 1,000 seats
# 50 seats  2 trips  6 days = 600 seats
Now, the operating cost can be found out as under:
Monthly Operating Cost Statement
Variable Costs Rs. Rs.
Diesel [(8,040 km  4 km)  Rs. 6] 12,060
Lubricant oil [(8,040 km  100)  Rs. 10] 804

39
Total variable cost (A) 12,864
Maintenance costs (B) 1,000
Fixed costs
Salary of driver 2,800
Salary of conductor 2,200
Salary of part-time accountant 200
Insurance 400
Road tax 125
Permit fee 315
Depreciation (Rs. 6,00,000  20%)  12 10,000
Total fixed cost (C) 16,040
Total costs per month (A+B+C) 29,904
Profit and Passenger tax @ 100% of cost (Note below) 29,904
Total takings per month 59,808
Passenger km. per month 3,72,000
Rate per passenger km. = (Rs. 59,808  3,72,000) = Re. 0.1607741 say Re. 0.16.
Proposed fare to be charged per passenger km.
Delhi to Chandigarh = Re. 0.16  150 km = Rs. 24.00
Delhi to Agra = Re. 0.16  120 km = Rs. 19.20
Delhi to Jaipur = Re. 0.16  270 km = Rs. 43.20
Note:
1. Passenger tax = 20% of total takings
2. Profit = 30% of total takings
3. Passenger tax + Profit = 50% of total takings
4. Cost (balance) = 50%
5. Total takings = 100%
6. Item 3 as a % of item 4 = 100%
Passenger tax (20% of Rs. 59,808) = Rs. 11,961.60
Profit (30% of Rs. 59,808) = Rs. 17,942.40
Rs. 29,904.00

Marginal Costing : Break Even Point, P/V Ratio and Margin of Safety
F 30,000
16. (i) BE point    Rs. 75,000
P/V ratio 40%

40
S  V 1,00,000  60,000
(ii) P/V ratio   40%
S 1,00,000
(iii) Margin of safety = Actual Sales – BE point
= 1,00,000 – 75,000 = Rs. 25,000
(iv) Break even chart showing contribution is shown below:

Break-even chart

Marginal Costing : Income Statement According to Marginal & Absorption Costing


17. (i) Income Statement (Marginal Costing)
August September
Rs. Rs.
(A) Sales 84,00,000 1,24,80,000
Variable manufacturing cost 50,00,000 40,00,000
Add: opening inventory @ Rs.10,000 per unit − 1,500,000
Cost of Goods available for sale 50,00,000 55,00,000

41
Less: Closing inventory @ Rs.10,000 per Unit 15,00,000 3,00,000
Variable cost of goods sold 35,00,000 52,00,000
Variable distribution cost 10,50,000 15,60,000
(B) Total variable cost 45,50,000 67,60,000
(C) Contribution (A-B) 38,50,000 57,20,000
Fixed cost:-Manufacturing 22,00,000 22,00,000
Marketing 4,00,000 4,00,000
(D) Total fixed cost 26,00,000 26,00,000
Net Income (C-D) 12,50,000 31,20,000
Income Statement (Absorption Costing)
August September
(A) Sales 84,00,000 1,24,80,000
Variable manufacturing 50,00,000 40,00,000
Fixed manufacturing cost 22,00,000 22,00,000
72,00,000 62,00,000
Add: opening inventory* − 21,00,000
Cost of goods available for sales 72,00,000 83,00,000
Less: Closing inventory* 21,60,000 4,65,000
Cost of goods sold (B) 50,40,000 78,35,000
Add: Distribution cost-variable 10,50,000 15,60,000
Add: Marketing cost-fixed 4,00,000 4,00,000
(B) Total Cost 64,90,000 97,95,000
Net Income (A-B) 19,10,000 26,85,000

(ii) Comments Marginal costing rewards sales while absorption costing rewards
production. This means that when sales are more then production, marginal costing
produces higher profit and vice versa, when production is more than sales,
absorption costing shows higher profit.
In August, absorption costing shows higher profit by Rs.6,60,000 (i.e., 19,10,000 –
12,50,000) than marginal costing because production is more than sales . In
September marginal costing shows higher profit than absorption costing by Rs.
4,35,000.
Sales are more than production. Difference in profit is exactly equal to difference in
inventory values in the two months.

42
*Working Notes:
In marginal costing inventory is valued at variable manufacturing cost while in absorption
costing inventory valuation is done as follows:
For September closing inventory of 30 Units:
Rs.
Variable manufacturing cost ( 30 units @ Rs.10,000) 3,00,000
Fixed manufacturing cost (30 units @ Rs.5,500) 1,65,000
4,65,000
Fixed manufacturing cost per unit is calculated as under
Rs. 22,00,000
 Rs. 5,500 per unit
400 units of production
For March, inventory of 150 units:
Rs.
Variable manufacturing cost (150 units @ Rs.10,000) 15,00,000
Fixed manufacturing cost (150 units @ Rs.4,400) 6,60,000
21,60,000

Rs. 22,00,000
Fixed manufacturing cost per unit   Rs. 4,400 per unit .
500 units of production

Standard Costing : Material and Labour Variance


18. For Material Cost Variances
Actual cost of material used (AQ × AR)
A 12,500 units × Rs.4.40 = Rs. 55,000
B 18,000 units × Rs.2.80 = Rs. 50,400
C 88,500 units × Rs.1.20 = Rs.1,06,200
2,11,600
Standard cost of material used (AQ × SR)
A 12,500 units × Rs.4.00 = Rs. 50,000
B 18,000 units × Rs.3.00 = Rs. 54,000
C 88,500 units × Rs.1.00 = Rs. 88,500
1,92,500

43
Standard material cost of production : (SQ for actual production × SR)
= 6,000 units × Rs. 32
= Rs.1,92,000
Variances:
Material Price Variance = (Actual cost of material used – Standard cost of material
used)
= Rs. 2,11,600 – Rs.1,92,500
= Rs. 19,100
Material Usage Variance = (Standard cost of material used – Standard material cost of
production)
= Rs. 1,92,500 – Rs. 1,92,000
= Rs. 500 (A)
For Labour Cost Variance :
Actual wages paid to workers (AH × AR)
2,500 hrs × Rs.12 = Rs. 30,000
15,000 hrs × Rs. 8 = Rs. 1,20,000
1,50,000
Standard cost for actual hours (AH × SR) :
17,500 hrs × Rs.8 = Rs.1, 40,000
Standard labour cost of output achieved (Actual output × SR of labour per unit):
= 6,000 units × Rs.24 = Rs.1, 44,000
Variances :
Labour Rate Variance : (Actual wages paid to workers – Standard cost for
actual hours)
= Rs.1,50,000 – Rs.1,40,000
= Rs.10,000 (A)
Labour Efficiency Variance : (Standard cost for actual hours – Standard labour cost
of output achieved)
= Rs.1,40,000 – Rs.1,44,000
= Rs.4,000 (F)

44
Standard Costing : Material Variance
19.
Material Standard Actual
Qty Rate Amt. Qty Rate Amt.
Kg Rs. Rs. Kg Rs. Rs.
EXE 50 12 600 ? 15 ?
POP ? 15 ? 70 20 1400
For determining actual input of EXE
Suppose actual input of EXE in kg = X
Material Mix Variance  Std. cost of St. mix per kg.  St. cost of actual mix per kg.
 Actual quantity mix per kg.

 12x  1,050 
(-) 45  13.50 *   (x  70) kg.
 x  70 
(-) 45 = 13.50 ( X+70) – (12X+1,050)
(-) 45 = (13.50X+945) – (12X+1,050)
- 45 = 1.5X – 105
X = 40 kg
Thus actual input of EXE = 40 kg
*Note: St. cost of mix per kg. is taken as the average cost of EXE and PQPs.
(Rs. 12 + Rs. 15) ÷2 = Rs.13.50
For determining standard input of PQP
Suppose standard quantity of PQP = Y
Material uses variance = ( SQ-AQ) × SP
(-) Rs.300 = (Y kg. − 70 kg) × Rs.15
Y = 50
Thus standard quantity of PQP = 50 kg

Computation of variances
Material price Variance = ( SP-AP) × AQ
EXE = (12 – 15)  40 = 120 (A)
PQP = (15-20) × 70 = Rs.350 (A)

45
Material Uses Variance = ( SQ – AQ) × SP
EXE = (50-40) × 12 = Rs.120 (F)
PQP = ( It is given in the question) Rs.300 (A)
Material Cost variance = ( SC- AC)
EXE = (50 × 12) – ( 40 × 15) = Rs. Nil
PQP = (50 × 15) – ( 70 × 20) = Rs.650 (A)

Varification:

Material Cost Variance = Material Price Variance + Material Uses Variance


EXE = Rs.120 (A) + Rs. 120 (F)
= Nil
PQP = Rs. 350 (A) + Rs. 300(A)
= Rs. 650 (A)

Standard Costing : Overhead Variance


20. (i) For Variable Overhead Variance:
Actual variable overhead = Rs.12, 000
Standard variable overhead
for production : (Budgeted output × Std. variable overhead rate per unit)
= (12,000 ÷ 4,000) × 3,800 = Rs.11, 400
Variable overhead variance: (Actual variable overhead – Standard variable
overhead)
= Rs.12, 000 – Rs.11, 400
= 600 (A)
(ii) For Fixed Overhead Variance:
Actual fixed overhead incurred = Rs. 39, 000
Budgeted fixed overhead
for the period = Rs. 40, 000
Standard fixed overhead
for production: (Standard output for actual time × Standard
Fixed Overhead per unit)
= (Rs.40, 000 ÷ 4,000 units) × 3,800 units
= Rs.38, 000.

46
Variances:
(a) Fixed Overhead Expenditure Variances:
(Actual fixed overhead – Budgeted fixed overhead)
= Rs.39, 000 – Rs.40, 000 = 1,000 (F)
(b) Fixed Overhead Volume Variance:
(Budgeted fixed overhead – Standard fixed overhead)
= Rs.40, 000 – Rs.38, 000 = Rs.2, 000 (A)
(c) Fixed Overhead Variance :
(Actual fixed overhead – Standard fixed overhead_
= Rs.39, 000 – Rs.38, 000 = Rs.1, 000 (A)

Varifiaction:
Fixed Overhead Variance = Fixed Overhead Expenditure Variance + Fixed Overhead
Volume Variance)
Rs. 1,000 (A) = Rs. 1,000 (F) + Rs. 2,000 (A)

Budgetary Control : Functional and Flexible Budget


21. (i) The functional budgets are broadly grouped under the following heads :
(a) Physical budgets - Those budgets which contains information in terms of
physical units about sales, production etc. for example, quantity of sales,
quantity of production, inventories, and manpower budgets are physical
budgets.
(b) Cost budgets - Budgets which provides cost information in respect of
manufacturing, selling, administration etc. for example, manufacturing costs,
selling costs, administration cost, and research and development cost budgets
are cost budgets.
(c) Profit budgets - A budget which enables in the ascertainment of profit, for
example, sales budget, profit and loss budget, etc.
(d) Financial budgets - A budget which facilitates in ascertaining the financial
position of a concern, for example, cash budgets, capital expenditure budget,
budgeted balance sheet etc.

47
(ii) Flexible Budget/ For the period ….
80% 90% 100% 110%
Rs. Rs. Rs. Rs.
Sales 6,00,000 6,75,000 7,50,000 8,25,000
Administration Costs:
Office Salaries (fixed) 90,000 90,000 90,000 90,000
General expenses (2% of Sales) 12,000 13,500 15,000 16,500
Depreciation (fixed) 7,500 7,500 7,500 7,500
Rent and rates (fixed) 8,750 8,750 8,750 8,750
(A) Total Adm. Costs 1,18,250 1,19,750 1,21,250 1,22,750
Selling Costs :
Salaries (8% of sales) 48,000 54,000 60,000 66,000
Travelling expenses (2% of sales) 12,000 13,500 15,000 16,500
Sales office (1% of sales) 6,000 6,750 7,500 8,250
General expenses (1% of sales) 6,000 6,750 7,500 8,250
(B) Total Selling Costs 72,000 81,000 90,000 99,000
Distribution Costs :
Wages (fixed) 15,000 15,000 15,000 15,000
Rent (1% of sales) 6,000 6,750 7,500 8,250
Other expenses (4% of sales) 24,000 27,000 30,000 33,000
(C) Total Distribution Costs 45,000 48,750 52,500 56,250
Total Costs (A + B + C) 2,35,250 2,49,500 2,63,750 2,78,000

Note : All fixed costs have been assumed to remain unchanged even at 110% capacity.
However, in practice, fixed costs may change when capacity utilisation exceeds 100%.

48
PART II : FINANCIAL MANAGEMENT

1. Answer the following, supporting the same with reasoning/working notes:


(a) Ram has deposited Rs. 55,650 in a bank, which is paying 15 per cent rate of
interest on a ten-year time deposit. Calculate the amount at the end of ten years?
(b) Calculate the amount which Mr. Shyam should deposit now to receive Rs. 50,000
after 15 years. The interest rate is 9 per cent.
(c) Which of the following statements concerning Net Operating Income (NOI) is
correct?
(i) That debt and equity levels remain unchanged.
(ii) That dividends increase at a constant rate.
(iii) That ko remains constant regardless of changes in leverage.
(iv) That interest expense and taxes are included in the calculation.
(d) Mr. X is considering purchasing a 12 year, 10% Rs. 100 par value preference share.
The redemption value of the preference share on maturity is Rs. 120. Mr. X’s
required rate of return is 10.5 per cent. What should he be willing to pay for the
share now?
(e) PL Forgings Limited estimates its total cash requirement as Rs. 2 crore next year.
The company’s opportunity cost of funds is 15 per cent per annum. It will have to
incur Rs. 150 per transaction when it converts its short-term securities to cash.
Calculate the optimum cash balance. Also, determine the total annual cost of the
demand for the optimum cash balance?
(f) The shares of Delta Limited are selling at Rs. 20 per share. The company had paid
dividend @ Rs. 2 per share last year. The estimated growth rate of the company is
approximately 5 per cent per year.
(i) Calculate the cost of equity capital.
(ii) Also, compute the estimated market price of the equity shares if the
anticipated growth rate of the company rises to 8 per cent or falls to 3 per cent.
(g) The decision criterion for the payback period technique is to accept the project if the
payback period is:
(i) Greater than the minimum time period acceptable to the firm.
(ii) Greater than the minimum time period acceptable for similar projects.
(iii) Less than one year.
(iv) None of the above.

49
(h) Alpha Limited has Rs. 10,00,000 in fixed costs. The company produces only one
product and anticipates selling each unit for Rs. 25 with variable costs of Rs. 5 per
unit. Calculate the break-even (quantity) point.
2. Financial Analysis and Planning
The following information relates to Gamma Trading Limited for the year ended 30th
June, 2008:
Gamma Trading Limited
Rs. Rs.
Sales 15,00,000 Administrative expenses:
Purchases 9,66,750 Salaries 81,000
Opening stock 2,28,750 Rent 8,100
Closing stock 2,95,500 Stationery, postage, etc. 7,500
Sales returns 60,000 Depreciation 27,900
Selling and distribution expenses Other charges 49,500
Salaries 45,900 Provision for taxation 1,20,000
Advertising 14,100 Non-operating income
Travelling 6,000 Dividend on shares 27,000
Non-operating expenses Profit on sale of shares 9,000
Loss on sale of assets 12,000
You are required:
(a) To rearrange the figures of Gamma Trading Company in a suitable form for
analysis, and
(b) Determine the following ratios:
(i) Gross profit ratio;
(ii) Operating stock; and
(iii) Stock turnover ratio.
3. Financing Decisions
The following information is given for Beta Limited:
Rs. in lakhs
EBIT 1,120
PBT 320
Fixed cost 700

50
You are required to compute:
(a) Degree of operating leverage
(b) Degree of financial leverage
(c) Degree of combined leverage
(d) Percentage change in earnings per share if sales increases by 5 per cent.
4. Management of Working Capital
Alcatech Limited has applied for a loan to Srinagar bank for financing its working capital
requirements. The bank has requested you to prepare an estimate of the requirements
of the working capital for the company. Add 10 percent to your estimated figure to cover
unforeseen contingencies. The information about the projected profit and loss account of
Alcatech is as follows:
(Rs.)
Sales 21,00,000
Cost of goods sold 15,30,000
Gross Profit 5,70,000
Less: Administrative 1,40,000
expenses
Selling expenses 1,30,000 2,70,000
Profit before tax 3,00,000
Provision for tax 1,00,000
Cost of goods sold has been derived as follows:
Material used 8,40,000
Wages and manufacturing expenses 6,25,000
Depreciation 2,35,000
17,00,000
Less: Stock of finished goods (10 percent not yet sold) 1,70,000
15,30,000

The figures given relate only to the goods that have been finished and not to work-in-
progress; goods equal to 15 percent of the year's production (in terms of physical units)
are in progress on an average, requiring full materials but only 40 percent of other
expenses. Alcatech believes in keeping two months consumption of material in stock;
desired cash balance is Rs. 40,000. Average time-lag in payment of all expenses is 1
month; suppliers of materials extend 1.5 months credit; sales are 20 percent cash; rest is
at two months credit; 70 percent of the income tax has to be paid in advance in quarterly
installments.

51
5. Management of Working Capital
Mr. Gupta, a customer of Goodluck Company, has been ordering 5,000 units at the rate
of 1000 units per order during last year. The production cost is Rs. 12 per unit - Rs. 8 for
materials and labour and Rs. 4 overhead cost. It costs Rs. 1,500 to set up for one run of
1,000 units and inventory-carrying cost is 20% of the production cost. Since Mr. Gupta
may buy at least 5,000 units this year, the company would like to avoid making five
different production runs. Calculate the most economic production run.
6. Management of Working Capital
Mahalaxmi Limited has budgeted its sales to be Rs. 7,00,000 per annum. Its costs as a
percentage of sales are as follows:
%
Raw materials 20
Direct labour 35
Overheads 15

Raw materials are carried in stock for two weeks and finished goods are held in stock
before sale for three weeks. Production takes four weeks. Mahalaxmi Limited takes four
weeks’ credit from suppliers and gives eight weeks’ credit to its customers. If both
overheads and production are incurred evenly throughout the year, what is Mahalaxmi
Limited’s total working capital requirement?
7. Financial Analysis and Planning
You are required to prepare a Cash Flow Statement according to (a) Direct Method (b)
Indirect Method as per AS-3 (Revised) from the following information:
Balance Sheet
as on 31.12. 2008
(Rs. in ‘000)
2008 2007
Assets
Cash on hand and balances with banks 200 25
Short-term investments 670 135
Sundry debtors 1,700 1,200
Interest receivable 100 --
Inventories 900 1,950
Long-term investments 2,500 2,500

52
Fixed assets at cost 2,180 1,910
Less: Accumulated depreciation (1,450) (1,060)
Fixed assets (net) 730 850
Total Assets 6,800 6,660

2008 2007
Liabilities
Sundry creditors 150 1,890
Interest payable 230 100
Income taxes payable 400 1,000
Long-term debt 1,110 1,040
Total liabilities 1,890 4,030
Shareholders’ funds
Share capital 1,500 1,250
Reserves 3,410 1,380
Total shareholders’ funds 4,910 2,630
Total Liabilities and Shareholders’ funds 6,800 6,660

Statement of Profit and Loss


for the period ended 31.12. 2008
(Rs. in ‘000)
Sales 30,650
Cost of sales (26,000)
Gross profit 4,650
Depreciation (450)
Administrative and selling expenses (910)
Interest expense (400)
Interest income 300
Dividend income 200
Foreign exchange loss (40)
Net profit before taxation and extraordinary item 3,350
Extraordinary item-

53
Insurance proceeds from earthquake disaster settlement 180
Net profit after extraordinary item 3,530
Income tax (300)
Net Profit 3,230

Additional Information: (Figures in Rs. ‘000)


(a) An amount of 250 was raised from the issue of share capital and a further 250 was
raised from long-term borrowings.
(b) Interest expense was 400 of which 170 was paid during the period. 100 relating to
interest expense of the prior period was also paid during the period.
(c) Dividends paid were 1,200.
(d) Tax deducted at source on dividends received (included in the tax expense of 300
for the year) amounted to 40.
(e) During the period, the enterprise acquired fixed assets for 350. The payment was
made in cash.
(f) Plant with original cost of 80 and accumulated depreciation of 60 was sold for 20.
(g) Foreign exchange loss of 40 represents the reduction in the carrying amount of a
short-term investment in foreign currency designated bonds arising out of a change
in exchange rate between the date of acquisition of the investment and the balance
sheet date.
(h) Sundry debtors and sundry creditors include amounts relating to credit sales and
credit purchases only.
8. Investment Decisions
Ganesha Enterprises is considering a proposal of installing a dust collector equipment.
The equipment would involve a cash outlay of Rs. 6,00,000 and net working capital of
Rs. 80,000. The expected life of the project is 5 years without any salvage value.
Assume that the company is allowed to charge depreciation on straight-line basis for
Income-tax purpose. The estimated before-tax cash inflows are given below:
Before-tax Cash inflows (Rs. ‘000)
Year 1 2 3 4 5
240 275 210 180 160
The applicable Income-tax rate to the company is 35%. If the company’s opportunity
cost of capital is 12%, calculate the equipment’s discounted payback period, payback
period, net present value and internal rate of return.

54
The PV factors at 12%, 14% and 15% are:
Year 1 2 3 4 5
PV factor at 12% 0.8929 0.7972 0.7118 0.6355 0.5674
PV factor at 14% 0.8772 0.7695 0.6750 0.5921 0.5194
PV factor at 15% 0.8696 0.7561 0.6575 0.5718 0.4972

9. Management of Working Capital


The present credit terms of Roltas Limited are 1/10 net 30. Its annual sales are Rs. 80
lakhs, its average collection period is 20 days. Its variable costs and average total costs
to sales are 0.85 and 0.95 respectively and its cost of capital is 10 per cent. The
proportion of sales on which customers currently take discount is 0.5. Roltas Limited is
considering relaxing its discount terms to 2/10 net 30. Such relaxation is expected to
increase sales by Rs. 5 lakhs, reduce the average collection period to 14 days and
increase the proportion of discount sales to 0.8. What will be the effect of relaxing the
discount policy on Roltas Limited’s profit? Take year as 360 days.
10. Investment Decisions
Best Luck Company is considering building an assembly plant and the company has two
options, out of which it wishes to choose the best plant. The projected output is 10,000
units per month. The following data are available:
Rs.
Plant A Plant B
Initial Cost 60,00,000 44,00,000
Direct Labour cost p.a. (1st Shift) 30,00,000 15,00,000
(2nd Shift) - 19,00,000
Overhead (per year) 5,00,000 4,20,000

Both the plants have an expected life of 10 years after which there will be no salvage
value. The cost of capital is 10%. The present value of an ordinary annuity of Re. 1 for
10 years @ 10% is 6.1446. Ignore effect of taxation.
You are required to determine:
(a) What would be the desirable choice?
(b) What other important elements are to be considered before the final decision is
taken?

55
11 (a) Scope and Objectives of Financial Management
Explain as to how the wealth maximisation objective is superior to the profit
maximisation objective.
(b) Financing Decisions
Explain, briefly, Modigliani and Miller approach on Cost of Capital
12. Scope and Objectives of Financial Management
“Modern financial management has come a long way from the traditional corporate
finance. As the economy is opening up and global resources are being tapped, the
opportunities available to finance managers virtually have no limits.” In view of these
changes, discuss the role of a chief financial officer (CFO).
13. Types of Financing
What is Venture Capital Financing? Enumerate the common methods of venture capital
financing.
14. Differentiate between the following:
(a) Bill Discounting and Bill Rediscounting
(b) Line of Credit and Letter of Credit
(c) Pledging and Factoring
(d) Weighted Average Cost of Capital (WACC) and Marginal Cost of Capital.
15. Write short notes on the following:
(a) Benefits of Commercial Papers
(b) Forms of Bank Credit
(c) Ageing Schedule of Monitoring Receivables
(d) Limitations of Financial Ratios.

SUGGESTED ANSWERS/HINTS

1 (a) Calculation of Future Value (FV)


FV = P  CVF10,0.15
FV = 55,650  CVF10,0.15
(CVF of Re. 1 at 15 percent for 10 years is 4.046)
= 55,650  4.046
= Rs. 2,25,159.90.

56
(b) PV = Amount  PVF15,0.09
(PVF at 9 percent for 15 years is 0.275)
PV = 50,000  PVF15,0.09
= 50,000  0.275
= Rs. 13,750
(c) The correct answer is (iii) because a critical assumption of the net operating income
(NOI) approach to valuation is that k o should remain constant regardless of changes
in leverage.
(d) Mr. X expects to receive Rs. 10 as preference dividend each year for 12 years and
Rs. 110 on maturity (i.e. at the end of 12 years). Using the present value annuity
factor to value the constant stream of preference dividends and the present value
factor to value the redemption payment, we can calculate the amount he should pay
now:
 1 1  120
P0  10    12   12
 0.105 0.105  (1.105)  (1.105)
= 10  6.506 + 120  0.302
= 65.06 + 36.24 = Rs. 101.30.
It can be observed that the present value of Rs. 101.30 is a sum of the present
value of dividends ( Rs. 65.06) and the present value of the redemption value ( Rs.
36.24). The Rs. 100 preference share is worth Rs. 101.3 today at 10.5 per cent
required rate of return.
Therefore, Mr. X should purchase the share for Rs. 100 now.
(e) Calculation of Optimum Cash Balance
2cT
C* 
k
2(150) (2,00,00,000)
C*   Rs. 2,00,000
0.15
Calculation of Total Annual Cost of Demand
The annual cost will be:
Total cost = 150 (2,00,00,000/2,00,000) + 0.15 (2,00,000/2)
= 150 (100) + 0.15 (1,00,000) = 15,000 + 15,000
= Rs. 30,000

57
(f) (i) Calculation of Cost of Equity Capital
ke = (D1 / P0) + g
= (Rs. 2.10 / Rs. 20) + 0.05
= 15.5 per cent
(ii) Calculation of the Market Price of Equity Shares
When growth rate rises to 8 percent
P0 = D / (ke – g)
= Rs. 2.16 / Rs. 0.075
= Rs. 28.80
When growth rate falls to 3 percent
P0 = D / [ke – (– g)]
P0 = D / (ke + g)
P0 = Rs. 1.94 / 0.185 = Rs. 10.49.
(g) The correct answer is (iv) because the decision criterion for the payback period
technique is to accept the project if the payback period is less than the minimum
time period acceptable for similar projects.
(h) Calculation of Break Even Point (BEP)

10,00,000
Break Even Point (BEP) 
(25 - 5)

10,00,000

20
BEP = 50,000
2. (a) Gross profit ratio
Rs. 6,00,000
  0.40 or 40%
Rs. 15,00,000
(b) Operating ratio
Cost of goods  Operating expenses

Sales
Rs. 11,40,000
  0.76 or 76%
Rs. 15,00,000

58
(c) Stock turnover ratio
Cost of goods sold

Average stock
Rs. 9,00,000
  3.43 times
Rs. 2,62,125
Where,
Opening Stock  Closing Stock
Average Stock 
2
2,28,750  2,95,500 5,24,250
  = 2,62,125
2 2
Gamma Trading Limited
Profit and Loss Statement
Rs.
Sales (less return) 15,00,000
Less: Cost of goods sold:
Opening stock 2,28,750
Purchases 9,66,750
11,95,500
Less: Closing stock 2,95,500 9,00,000
Gross profit 6,00,000
Operating expenses
Selling and distribution expenses 66,000
Administrative expenses 1,74,000 2,40,000
Operating net profit 3,60,000
Non – operating income 36,000
Non - operating expenses 12,000 24,000
Profit before tax 3,84,000
Provision for taxes 1,20,000
2,64,000

3. (a) Degree of Operating Leverage (DOL)

Contribution EBIT  Fixed Cost 1,120  700


DOL     1.625
EBIT EBIT 1,120

59
(b) Degree of Financial Leverage (DFL)
EBIT
DFL 
PBT
1,120
  3.5
320
(c) Degree of Combined Leverage (DCL)
DCL = DOL  DFL
= 1.625  3.5
= 5.6875
(d) Percentage change in EPS if sales increases by 5 percent
% Change in EPS
DCL 
% Change in Sales
% Change in EPS
5.6875 
5
% change in EPS = 5  5.6875
= 28.4375%.

4. Estimate of Net Working Capital of Alcatech Limited


(A) Current Assets: (Rs.)
(i)  2 1,40,000
Raw material in stock  Rs. 8,40,000  
 12 
(ii) Work-in-progress
(a) Raw material (Rs. 8,40,000  15/100) 1,26,000
(b) Wages and manufacturing expenses 37,500
(Rs. 6,25,000  0.4  15/100)
(iii) Stock of finished goods
[Rs. 1,70,000  Rs. 23,500 (0.10  Rs. 1,46,500
2,35,000; depreciation)]
(iv) Debtors
(a) Cost of goods sold Rs. 15,30,000
Less: Depreciation
(Rs. 2,35,0000.9) 2,11,500

60
13,18,500
(b) Administrative expenses 1,40,000
(c) Selling expenses 1,30,000
Total 15,88,500
Credit sales (4/5 of Rs. 15,88,500 = 12,70,800 2,11,800
 2
 12,70,800  
 12 
(v) Cash required 40,000
Total Investment in Current Assets 7,01,800
(B) Current Liabilities:
(i) Average time-lag in payment of expenses:
(a) Wages and manufacturing expenses 6,25,000
(b) Administrative expenses 1,40,000

(c) Selling expenses 1,30,000


8,95,00012 74,583
(ii) Creditors (Rs. 8,40,0003/24) 1,05,000
Total Current Liabilities 1,79,583

(C) Net Working Capital: Current Assets  Current 5,22,217


Liabilities
Add: 10 percent contingencies 52,222
5,74,439

Assumptions and Working Notes:


(a) Depreciation is not a cash expense and, therefore, excluded from cost of goods sold
for the purpose of determining work-in-progress, finished goods and investment in
debtors.
(b) Since profit is not taken into consideration in the calculation as a source of working
capital, income tax has been excluded as it is to be paid out of profits.
5. The most economic production run is:

2  5,000  1,500
=
12  20%

61
10,000 1,500
=
2.40

1,50,00,000
=
2.40

= 62,50,000

= 2,500 units.
Therefore, the most economic production run is of 2,500 units.
6. Annual costs:
Raw materials: 7,00,0000.20 = Rs. 1,40,000
Direct labour: 7,00,0000.35 = Rs. 2,45,000
Overheads: 7,00,0000.15 = Rs. 1,05,000

Calculation of Working Capital Requirement


Rs. Rs.
Stock of raw materials: 1,40,000  (2/52) 5,385
Work-in-progress:
Materials : 1,40,000  (4/52) 10,769
Labour : 2,45,000  (4/52)  ½ 9,423
Overheads : 1,05,000  (4/52)  ½ 4,038
24,230
Finished goods : 4,90,000  (3/52) 28,269
Debtors : 7,00,000  (8/52) 1,07,692
Creditors : 1,40,000  (4/52) (10,769)
Working Capital Required 1,54,807

Note: Work-in-progress is assumed to be half complete as regards labour and


overheads, but fully complete as regards raw materials, i.e. all raw materials are added
at the start of production.

62
7. CASH FLOW STATEMENT
(Direct Method)
(Rs. in ‘000)
2008
Cash flows from Operating Activities
Cash receipts from customers 30,150
Cash paid to suppliers and employees (27,600)
Cash generated from operations 2,550
Income taxes paid (860)
Cash flow before extraordinary item 1,690
Proceeds from earthquake disaster settlement 180
Net cash from operating activities 1,870
Cash flows from Investing Activities
Purchase of fixed assets (350)
Proceeds from sale of equipment 20
Interest received 200
Dividend received 160
Net cash from investing activities 30
Cash Flows from Financing Activities
Proceeds from issuance of share capital 250
Proceeds from long-term borrowings 250
Repayments of long-term borrowings (180)
Interest paid (270)
Dividend paid (1,200)
Net cash used in financing activities (1,150)
Net increase in Cash and Cash Equivalents 750
Cash and Cash Equivalents at beginning of period (See Note 1) 160
Cash and Cash Equivalents at end of period (See Note 1) 910

63
Notes to the Cash Flow Statement (Direct & Indirect Method)
1. Cash and Cash Equivalents: Cash and cash equivalents consist of cash on hand and
balance with banks, and investments in money-market instruments. Cash and cash
equivalents included in the cash flow statement comprise the following balance sheet
amounts.
2008 2007
Cash on hand and balance with banks 200 25
Short-term investments 670 135
Cash and cash equivalents 870 160
Effects of exchange rate changes 40 --
Cash and cash equivalents as restated 910 160

Cash and cash equivalents at the end of the period include deposits with banks of 100
held by a branch which are not freely permissible to the company because of currency
exchange restrictions.
The company has undrawn borrowing facilities of 2,000 of which 700 may be used only
for future expansion.
2. Total tax paid during the year (including tax deducted at source on dividends received)
amounted to 900.
CASH FLOW STATEMENT
(Indirect Method)
(Rs. in ‘000)
2008
Cash flows from operating activities
Net profit before taxation, and extraordinary item 3,350
Adjustments for:
Depreciation 450
Foreign exchange loss 40
Interest income (300)
Dividend income (200)
Interest expense 400
Operating profit before working capital changes 3,740
Increase in sundry debtors (500)

64
Decrease in inventories 1,050
Decrease in sundry creditors (1,740)
Cash generated from operations 2,550
Income taxes paid (860)
Cash flows before extraordinary item 1,690
Proceeds from earthquake disaster settlement 180
Net cash from operating activities 1,870
Cash flows from investing activities
Purchase of fixed assets (350)
Proceeds from sale of equipment 20
Interest received 200
Dividends received 160
Net cash from investing activities 30
Cash flows from financing activities
Proceeds from issuance of share capital 250
Proceeds from long-term borrowings 250
Repayment of long-term borrowings (180)
Interest paid (270)
Dividends paid (1,200)
Net cash used in financing activities (1,150)
Net increase in cash and cash equivalents 750
Cash and cash equivalents at beginning of period (See Note 1) 160
Cash and cash equivalents at end of period (See Note 1) 910

Alternative Presentation (Indirect Method)


As an alternative, in an indirect method cash flow statement, operating profit before
working capital changes is sometimes presented as follows:
Revenues excluding investment income 30,650
Operating expenses excluding depreciation (26,910)
Operating profit before working capital changes 3,740

65
Working Notes:
The working notes given below do not form part of the cash flow statement. The purpose
of these working notes is merely to assist in understanding the manner in which various
figures in the cash flow statement have been derived.
(Figures are in Rs.’000)
1. Cash receipts from customers
Sales 30,650
Add: Sundry debtors at the beginning of the year 1,200
31,850
Less: Sundry debtors at the end of the year 1,700
30,150
2. Cash paid to suppliers and employees
Cost of sales 26,000
Administrative & selling expenses 910
26,910

Add: Sundry creditors at the beginning of the year 1,890


Inventories at the end of the year 900 2,790
29,700
Less: Sundry creditors at the end of the year 150
Inventories at the beginning of the year 1,950 2,100
27,600

3. Income taxes paid (including tax deducted at source from


dividends received)
Income tax expense for the year 300
(including tax deducted at source from dividends received
Add: Income tax liability at the beginning of the year 1,000
1,300
Less: Income tax liability at the end of the year 400
900
Out of 900, tax deducted at source on dividends received
(amounting to 40), is included in cash flows from investing
activities and the balance of 860 is included in cash flows
from operating activities.
4. Repayment of long-term borrowings
Long-term debt at the beginning of the year 1,040

66
Add: Long-term borrowings made during the year 250
1,290
Less: Long-term borrowings at the end of the year 1,110
180
5. Interest paid
Interest expense for the year 400
Add: Interest payable at the beginning of the year 100
500
Less: Interest payable at the end of the year 230
270

8. (i) Equipment’s initial cost = Rs. 6,00,000 + 80,000 = Rs. 6,80,000


(ii) Annual straight line depreciation = Rs. 6,00,000/5 = Rs. 1,20,000
(iii) Net cash flows can be calculated as follows:
= Before tax CFs × (1 – Tc) + Tc × Depreciation
(Rs. ‘000)
Cash flows
Year 0 1 2 3 4 5
Initial cost (680)
Before tax cash flows 240 275 210 180 160
Tax @ 35% 84 96.25 73.5 63 56
After tax cash flows 156 178.75 136.5 117 104
Depreciation tax shield
(Depreciation × Tc) 42 42 42 42 42
Working capital released − − − − 80
Net Cash Flow 198 220.75 178.5 159 226
PVF at 12% 1.00 0. 8929 0.7972 0.7118 0.6355 0.5674
PV (680) 176.79 175.98 127.06 101.04 128.24
NPV 29.12

0 1 2 3 4 5
PVF at 15% 1 0.8696 0.7561 0.6575 0.5718 0.4972
PV (680) 172.18 166.91 117.36 90.92 112.37
NPV (20.26)

Internal Rate of Return


29.12
IRR  12%   3% = 13.77%
49.38

67
Discounted Payback Period
Discounted Cash flows at K = 12% considered
99.13
= 176.79 + 175.98 + 127.06 + 101.04 + 12 ×
128.24
= 4 years and 9.28 months
Payback Period (Net Cash flows are considered)
82.75
= 198 + 220.75 + 178.5 + 12 ×
159
= 3 years and 6.25 months
9. Working Notes:
Calculation of Reduction in Investment in Receivables (Rs.)
Present investment in receivables
 20 days  4,22,222
Rs. 80 lakhs  0.95  
 360 days 
Proposed investment in receivables
 14 days  3,12,083
{(Rs. 80 lakhs  0.95)  (Rs. 5 lakhs  0.85)}  360 days 
 
Reduction in investment in receivables 1,10,139
Calculation of Increase in Discount (Rs.)
Present discount (Rs. 80 lakhs  1/100  0.5) 40,000
Proposed discount (Rs. 85 lakhs  2/100  0.8) 1,36,000
Net increase in discount 96,000
Statement Showing Evaluation of Effect of Relaxing the Discount Policy on Roltas
Limited’s Profit
(Rs.)
Incremental Revenue:
Increase in contribution (Rs. 5 lakhs 15/100) 75,000
Cost of savings on investment in receivables (Rs. 1,10,139 lakhs  11,014
10/100)
Total Incremental Cost Total (A) 86,014

68
Net increase in discount (B) 96,000
Incremental Loss 9,986

Analysis: There will be an incremental loss by relaxing the discount policy. Hence, it is
not suggested to release the Roltas Limited’s present discount policy.
10. (A) Computation of Differential Cash flow
(Rs.)
Plant A Plant B Differential cash
outflow
Direct labour cost:
1st shift 30,00,000 15,00,000 (15,00,000)
2nd shift - 19,00,000 19,00,000
Overhead 5,00,000 4,20,000 (80,000)
Net saving for using Plant A 3,20,000

Present value of net saving of (Rs. 3,20,0006.1446) = Rs. 19,66,272. Plant A @


10% (cost of capital)
(B) Additional Cash Outlay for Plant A over Plant B
Rs.
Cost of Plant A 60,00,000
Cost of Plant B 44,00,000
Additional outlay for using Plant A 16,00,000

Analysis: The net saving for the company in choosing plant A = Rs. 19,66,272 –
Rs. 16,00,000 = Rs. 3,66,272. Hence, Plant A should be implemented.
11 (a). Superiority of Wealth Maximisation Objective to Profit Maximisation Objective
A firm’s financial management may often have the following as their objectives:
(i) The maximisation of firm’s profit.
(ii) The maximisation of firm’s value / wealth.
The maximisation of profit is often considered as an implied objective of a firm. To
achieve the aforesaid objective various type of financing decisions may be taken.
Options resulting into maximisation of profit may be selected by the firm’s decision
makers. They even sometime may adopt policies yielding exorbitant profits in short
run which may prove to be unhealthy for the growth, survival and overall interests of

69
the firm. The profit of the firm in this case is measured in terms of its total
accounting profit available to its shareholders.
The value/wealth of a firm is defined as the market price of the firm’s stock. The
market price of a firm’s stock represents the focal judgment of all market
participants as to what the value of the particular firm is. It takes into account
present and prospective future earnings per share, the timing and risk of these
earnings, the dividend policy of the firm and many other factors that bear upon the
market price of the stock.
The value maximisation objective of a firm is superior to its profit maximisation
objective due to the following reasons:
(i) The value maximisation objective of a firm considers all future cash flows,
dividends, earning per share, risk of a decision etc. whereas profit
maximisation objective does not consider the effect of earning per share,
dividend paid or any other returns to shareholders or the wealth of the
shareholder.
(ii) A firm that wishes to maximise the shareholders wealth may pay regular
dividends whereas a firm with the objective of profit maximisation may refrain
from dividend payment to its shareholders.
(iii) Shareholders would prefer an increase in the firm’s wealth against its
generation of increasing flow of profits.
(iv) The market price of a share reflects the shareholders expected return,
considering the long-term prospects of the firm, reflects the differences in
timings of the returns, considers risk and recognizes the importance of
distribution of returns.
The maximisation of a firm’s value as reflected in the market price of a share is
viewed as a proper goal of a firm. The profit maximisation can be considered as a
part of the wealth maximisation strategy.
(b) Modigliani and Miller (MM) Approach to Cost of Capital
Modigliani and Miller’s argue that the total cost of capital of a particular corporation
is independent of its methods and level of financing. According to them a change in
the debt equity ratio does not affect the cost of capital. This is because a change in
the debt equity ratio changes the risk element of the company which in turn changes
the expectations of the shareholders from the particular shares of the company.
Hence they contend that leverages has little effect on the overall cost of capital or
on the market price.
Modigliani and Miller made the following assumptions and the derivations there
from:

70
Assumptions
(i) Capital markets are perfect. Information is costless and readily available to all
investors, there are no transaction costs; and all securities are infinitely
divisible. Investors are assumed to be rational and to behave accordingly.
(ii) The average expected future operating earnings of a firm are represented by a
subjective random variable. It is assumed that the expected values of the
probability distributions of all investors are the same. Implied in the MM
illustration is that the expected values of the probability distributions of
expected operating earnings for all future periods are the same as present
operating earnings.
(iii) Firms can be categorized into “equivalent return” classes. All firms within a
class have the same degree of business risk.
(iv) The absence of corporate income taxes is assumed.
Three Basic Propositions
(i) The total market value of the firm and its cost of capital are independent of its
capital structure. The total market value of a firm is given by capitalising the
expected stream of operating earnings at a discount rate appropriate for its risk
class.
(ii) The expected yield of a share of stock, K e is equal to the capitalisation rate of
a pure equity stream, plus a premium for financial risk equal to the difference
between the pure equity capitalization rate and Kg times the ratio B/S. In other
words, Ke increases in a manner to exactly offset the use of cheaper debt
funds.
(iii) The cut-off rate for investment purposes is completely independent of the way
in which an investment is financed. This proposition alongwith the first implies
a complete separation of the investment and financing decisions of the firm.
Thus, the theory propounded by Modigliani and Miller is based on the prevalence of
perfect market conditions which are rare to find. Corporate taxes and personal taxes
are a reality and they exert appreciable influence over decision making whether to
have debt or equity.
12. Role of Chief Financial Officer (CFO)
A new era has ushered during the recent years in financial management, especially with
the development of new financial system, emergence of financial services industry,
recent innovations and development of financial tools, techniques, instruments, and
products and emphasis on public sector undertakings to be self-supporting and their
dependence on capital market for fund requirements, have all changed the role of the
chief financial officer. His role, especially, assumes significance in the present day
context of liberalization, deregulation and globalisation.

71
Due to the changes in the global environment the chief financial officer needs to have a
broader and far-sighted outlook, and must realize that his actions would have far
reaching consequences for the firm because they influence the size, profitability, growth,
risk and survival of the firm, and as a consequence, affect the overall value of the firm.
He must have the flexibility to adapt to the external factors such as economic uncertainty,
global competition, technological change, volatility of interest and exchange rates,
changes in laws and regulations and ethical concerns. Therefore, in today’s changing
environment, the chief financial officer plays a pivotal leadership role in a company’s
overall efforts to achieve its goals.
He is one of the dynamic members of corporate managerial team. His role, day-by-day, is
becoming more and more pervasive and significant in solving the complex managerial
problems. The traditional role of the chief financial officer was confined just to raising of
funds from a number of sources, but the recent development in the socio-economic and
political scenario throughout the world has placed him in a central position in the
business organisation. He is now responsible for shaping the fortunes of the enterprise,
and is involved in the most vital decision of allocation of capital like mergers, acquisition
etc. He like other members of corporate team cannot be averse to the fast developments
around him. He has to take note of these changes in order to be relevant and dynamic
according to the fast changing circumstances.
To sum it up, the chief financial officer of an organisation plays an important role in the
company’s goals, policies, and financial success. His responsibilities include:
(a) Financial analysis and planning: Determining the proper amount of funds to employ
in the firm, i.e. designating the size of the firm and its rate of growth.
(b) Investment decisions: The efficient allocation of funds to specific assets.
(c) Financing and capital structure decisions: Raising funds on favourable terms as
possible, i.e., determining the composition of liabilities.
(d) Management of financial resources (such as working capital).
(e) Risk management: Protecting assets.
13. Venture Capital Financing and its Methods
The venture capital financing refers to financing of new high risky venture promoted by
qualified entrepreneurs who lack experience and funds to give shape to their ideas. In
broad sense, under venture capital financing venture capitalists make investment to
purchase equity or debt securities from inexperienced entrepreneurs who undertake
highly risky ventures with a potential of success.
Some common methods of venture capital financing are as follows:
(i) Equity financing: The venture capital undertakings generally require funds for a
longer period but may not be able to provide returns to the investors during the
initial stages. Therefore, the venture capital finance is generally provided by way of

72
equity share capital. The equity contribution of venture capital firm does not exceed
49% of the total equity capital of venture capital undertakings so that the effective
control and ownership remains with the entrepreneur.
(ii) Conditional loan: A conditional loan is repayable in the form of a royalty after the
venture is able to generate sales. No interest is paid on such loans. In India venture
capital financiers charge royalty ranging between 2 and 15 per cent; actual rate
depends on other factors of the venture such as gestation period, cash flow
patterns, risk and other factors of the enterprise. Some venture capital financiers
give a choice to the enterprise of paying a high rate of interest (which could be well
above 20 per cent) instead of royalty on sales once it becomes commercially sound.
(iii) Income note: It is a hybrid security which combines the features of both
conventional loan and conditional loan. The entrepreneur has to pay both interest
and royalty on sales but at substantially low rates. IDBI’s VCF provides funding
equal to 80 – 87.50% of the project’s cost for commercial application of indigenous
technology.
(iv) Participating debenture: Such security carries charges in three phases — in the
start up phase no interest is charged, next stage a low rate of interest is charged up
to a particular level of operation, after that, a high rate of interest is required to be
paid.
14. (a) Bill Discounting and Bill Rediscounting
Bill discounting is recognized as an important short term financial Instrument and it
is widely used method of short term financing. In a process of bill discounting, the
supplier of goods draws a bill of exchange with direction to the buyer to pay a
certain amount of money after a certain period, and gets its acceptance from the
buyer or drawee of the bill.
Whereas, Bill Rediscounting scheme, on the other hand, was introduced by Reserve
Bank of India with effect from 1st November, 1970 in order to extend the use of the
bill of exchange as an instrument for providing credit and the creation of a bill
market in India with a facility for the rediscounting of eligible bills by banks. Under
the bills rediscounting scheme, all licensed scheduled banks are eligible to offer
bills of exchange to the Reserve Bank for rediscount.
(b) Line of Credit and Letter of Credit
Line of Credit is a commitment by a bank to lend a certain amount of funds on
demand specifying the maximum amount.
Whereas, Letter of Credit, on the other hand, is an arrangement by which the
issuing bank on the instructions of a customer or on its own behalf undertakes to
pay or accept or negotiate or authorizes another bank to do so against stipulated
documents subject to compliance with specified terms and conditions.

73
(c) Pledging and Factoring
Pledging refers to the use of a firm’s receivable to secure a short term loan. A
firm’s receivables can be termed as its most liquid assets and these serve as prime
collateral for a secured loan. The lender scrutinizes the quality of the accounts
receivables, selects acceptable accounts, creates a lien on the collateral and fixes
the percentage of financing receivables which ranges around 50 to 90%. The major
advantage of pledging accounts receivables is the ease and flexibility it provides to
the borrower. Moreover, financing is done regularly. This, however, suffers on
account of high cost of financing.
Whereas, Factoring, on the other hand, is a new concept in financing of accounts
receivables. This refers to out right sale of accounts receivables to a factor or a
financial agency. A factor is a firm that acquires the receivables of other firms. The
factoring lays down the conditions of the sale in a factoring agreement. The
factoring agency bears the right of collection and services the accounts for a fee.
(d) Weighted Average Cost of Capital (WACC) and Marginal Cost of Capital
Weighted Average Cost of Capital represents the investors' opportunity cost of
taking on the risk of putting money into a company. Since every company has a
capital structure i.e. what percentage of debt comes from retained earnings, equity
shares, preference shares, and bonds, so by taking a weighted average, it can be
seen how much interest the company has to pay for every rupee it borrows. This is
the weighted average cost of capital. Thus, weighted average cost of capital is the
weighted average after tax costs of the individual components of firm’s capital
structure. That is, the after tax cost of each debt and equity is calculated separately
and added together to a single overall cost of capital.
Whereas, the Marginal Cost of Capital, on the other hand, may be defined as the
cost of raising an additional rupee of capital. Since the capital is raised in
substantial amount in practice marginal cost is referred to as the cost incurred in
raising new funds. Marginal cost of capital is derived, when the average cost of
capital is calculated using the marginal weights. The marginal weights represent the
proportion of funds the firm intends to employ. Thus, the problem of choosing
between the book value weights and the market value weights does not arise in the
case of marginal cost of capital computation. To calculate the marginal cost of
capital, the intended financing proportion should be applied as weights to marginal
component costs. The marginal cost of capital should, therefore, be calculated in
the composite sense. When a firm raises funds in proportional manner and the
component’s cost remains unchanged, there will be no difference between average
cost of capital (of the total funds) and the marginal cost of capital.

74
15. (a) Benefits of Commercial Papers (CP)
From the point of the issuing company, Commercial Papers provide the following
benefits:
(i) CP is sold on an unsecured basis and does not contain any restrictive
conditions.
(ii) Maturing CP can be repaid by selling new CP and thus can provide a
continuous source of funds.
(iii) Maturity of CP can be tailored to suit the requirement of the issuing firm.
(iv) CP can be issued as a source of fund even when money market is tight.
(v) Generally, the cost of CP to the issuing firm is lower than the cost of
commercial bank loans.
(b) Forms of Bank Credit
The bank credit will generally be in the following forms:
 Cash Credit: This facility will be given by the banker to the customers by giving
certain amount of credit facility on continuous basis. The borrower will not be
allowed to exceed the limits sanctioned by the bank.
 Bank Overdraft: It is a short-term borrowing facility made available to the
companies in case of urgent need of funds. The banks will impose limits on the
amount they can lend. When the borrowed funds are no longer required they can
quickly and easily be repaid. The banks issue overdrafts with a right to call them in
at short notice.
 Bills Discounting: The company which sells goods on credit, will normally draw a
bill on the buyer who will accept it and sends it to the seller of goods. The seller, in
turn discounts the bill with his banker. The banker will generally earmark the
discounting bill limit.
 Bills Acceptance: To obtain finance under this type of arrangement a company
draws a bill of exchange on bank. The bank accepts the bill thereby promising to
pay out the amount of the bill at some specified future date.
 Line of Credit: Line of Credit is a commitment by a bank to lend a certain amount
of funds on demand specifying the maximum amount.
 Letter of Credit: It is an arrangement by which the issuing bank on the
instructions of a customer or on its own behalf undertakes to pay or accept or
negotiate or authorizes another bank to do so against stipulated documents
subject to compliance with specified terms and conditions.

75
 Bank Guarantees: Bank guarantee is one of the facilities that the commercial
banks extend on behalf of their clients in favour of third parties who will be the
beneficiaries of the guarantees.
(c) Ageing Schedule of Monitoring Receivables
When receivables are analysed according to their age, the process is known as
preparing the ageing schedules of receivables. The computation of average age of
receivables is a quick and effective method of comparing the liquidity of receivables
with the liquidity of receivables in the past and also comparing liquidity of one firm
with the liquidity of the other competitive firm. It also helps the firm to predict
collection pattern of receivables in future. This comparison can be made
periodically. The purpose of classifying receivables by age groups is to have a
closer control over the quality of individual accounts. It requires going back to the
receivables ledger where the dates of each customer’s purchases and payments
are available. The ageing schedule, by indicating a tendency for old accounts to
accumulate, provides a useful supplement to average collection period of
receivables/sales analysis. Because an analysis of receivables in terms of
associated dates of sales enables the firm to recognise the recent increases, and
slumps in sales. To ascertain the condition of receivables for control purposes, it
may be considered desirable to compare the current ageing schedule with an earlier
ageing schedule in the same firm and also to compare this information with the
experience of other firms.
(d) Limitations of Financial Ratios
The limitations of financial ratios are listed below:
(i) Diversified product lines:Many businesses operate a large number of divisions in
quite different industries. In such cases ratios calculated on the basis of
aggregate data cannot be used for inter-firm comparisons.
(ii) Financial data are badly distorted by inflation: Historical cost values may be
substantially different from true values. Such distortions of financial data are
also carried in the financial ratios.
(iii) Seasonal factors may also influence financial data.
(iv) To give a good shape to the popularly used financial ratios (like current ratio,
debt- equity ratios, etc.): The business may make some year-end adjustments.
Such window dressing can change the character of financial ratios which would
be different had there been no such change.
(v) Differences in accounting policies and accounting period: It can make the
accounting data of two firms non-comparable as also the accounting ratios.
(vi) There is no standard set of ratios against which a firm’s ratios can be compared:
Some times a firm’s ratios are compared with the industry average. But if a firm

76
desires to be above the average, then industry average becomes a low
standard. On the other hand, for a below average firm, industry averages
become too high a standard to achieve.
(vii) It is very difficult to generalise whether a particular ratio is good or bad: For
example, a low current ratio may be said ‘bad’ from the point of view of low liquidity,
but a high current ratio may not be ‘good’ as this may result from inefficient
working capital management.
(viii) Financial ratios are inter-related, not independent: Viewed in isolation one ratio
may highlight efficiency. But when considered as a set of ratios they may speak
differently. Such interdependence among the ratios can be taken care of
through multivariate analysis.
Financial ratios provide clues but not conclusions. These are tools only in the
hands of experts because there is no standard ready-made interpretation of
financial ratios.

77

You might also like